0% found this document useful (0 votes)
358 views177 pages

Binder 1

Uploaded by

Chhim Tykoykinds
Copyright
© Attribution Non-Commercial (BY-NC)
We take content rights seriously. If you suspect this is your content, claim it here.
Available Formats
Download as PDF, TXT or read online on Scribd
0% found this document useful (0 votes)
358 views177 pages

Binder 1

Uploaded by

Chhim Tykoykinds
Copyright
© Attribution Non-Commercial (BY-NC)
We take content rights seriously. If you suspect this is your content, claim it here.
Available Formats
Download as PDF, TXT or read online on Scribd
You are on page 1/ 177

A

L
M

eroberogeday

lwm pln nig Esn Bisid

PaKTI 10
i

KNkmkarniBn nig eroberog


lwm pln nig Esn Bisid

KNkmkarRtYtBinitbeckeTs
elak
elak
elak
elak

lwm qun
nn; suxNa
RBwm sunit
Tit em:g

elak Gwug sMNag


elakRsI Tuy rINa
elak pl bunqay

KNkmkarRtYtBinitGkraviruT
elak lwm miKsir

karIkMuBTr
kBaa lI KuNaka

ii

elak Gwug sMNag

Garmfa
esoePAKNitviTaCMuvijBiPBelakPaKTI10 EdlGksikSakMBugkan;
enAkgdenH EckecjCabIEpkEdlEpkTI1 CakRmglMhat;eRCIserIs
EpkTI2 CaEpkdMeNaHRsay nig EpkTI3 CalMhat;Gnuvtn_ .
ral;bRbFanlMhat;nImYyenAkgesovePAenH eyIgxJM)aneRCIserIsykEt lMhat;
NaEdlmanlkNBi)ak mkeFVIdMeNaHRsayKMrUy:agek,aHk,aybMput .
eKalbMNgnkareroberogcgRkgKWedIm,ITukCaksarCMnYysRmab;Gk
sikSakgRKb;mCdan nig ma:geTotedIm,IcUlrYmelIksyvisyKNitviTakg
RbeTskm<CaeyIg[kan;EtrIkceRmInqab;rhsRsbtamsmyviTasaRsTMenIb.
esovePAenHminlhYseKhYsgenaHeT . kMhusqgedayGectnaR)akd
CamanTaMgbeckeTs nig GkraviruT . ehtuenHeyIgxJMCaGkniBn rgcaMCanicnUv
mtiriHKn;EbbsabnaBIGksikSaRKb;mCdanedaykIrIkrayedIm,IEklMGesovenH
[kan;EtmansuRkitPaBRbesIreLIgEfmeTot .
CaTIbBab; xJM)aTGkniBnsUmeKarBCUnBrGksikSaTaMgGs;mansuxPaBl
manR)aCJaQasv nig TTYl)aneCaKCyCanickgkarsikSa .
)at;dMbg fTI 27 Exmifuna qaM2011
GkniBn nig RsavRCav lwm pln
Tel : 017 768 246
Email: lim_phalkun@ymail.com
Website: www.mathtoday.wordpress.com

iii

KNitviTaCMuvijBiPBelak

eroberogeday lwm pln

- TMBr 1 -

KNitviTaCMuvijBiPBelak
CMBUkTI1

kRmglMhat;eRCIserIs
!> eK[cMnYnBit x1 , x2 , y1 , y 2 , z1 , z 2 epgpat; x1 0 , x2 0
x1y1 z12 0 nig x 2y 2 z 2 2 0 .
cUrRsayfa
8
1
1

( x1 x 2 )( y1 y 2 ) ( z1 z 2 )2 x1y1 z12 x 2y 2 z 22

( IMO 1969 )

@> eKtag I nig O erogKaCapitrgVg;carwkkgnigpitrgVg;carwkeRkA


nRtIekaN eK[RtIekaN ABC mYy .
cUrRsayfa OIA 90o luHRtaEt AB , BC , CA CasIVtnBVn .
#> eK[ n CacMnYnKt;viCmanedaydwgfa 2 2 28n2 1 CacMnYnKt;.
cUrRsayfa 2 2 28n2 1 CakaerR)akdncMnYnKt;mYy .
$> eKyk x CacMnYnBit edaydwgfa 0 x 2 .
cUrbgajfa cos2 (x) cot(x) sin 2 (x) tan( x) 1

( Baltic Way 2010 )

eroberogeday lwm pln

- TMBr 2 -

KNitviTaCMuvijBiPBelak
%> eK[RtIekaN ABC mYymanRCug a , b , c . tag r nig R erogKaCa
kaMrgVg;carwkkg nig kaMrgVg;carwkeRkAn ABC .
k> cURsayfa a cos A bosB c cos C 2Rpr
cos A cos B cos C a 2 b 2 c 2

a
b
c
2abc
r
cos A cos B cos C 1
R
abc
p
2
a 2 b 2 c 2 4 ( R r )2 A , B, C

CaknHbrimaRtnRtIekaN .
Edl
x> cUrTajbBaak;fa

CamMuRsYc.
^> eK[ f CaGnuKmn_kMNt;BIsMNMu IR eTAsMNMu IR Edlepgpat;
lkxN f (x y ) f (x).f (y ) cMeBaHRKb; x , y IR nig f '(0)
cUrkMNt;rkGnuKmn_ y f (x) .
21
&> sIVt{an } kMNt;eday a1 16
nigcMeBaH n 2 : 2a 3a 3
2
eKyk m CacMnYnKt;mYyEdl m 2 . cUrbgajfacMeBaH n m
*

eyIg)an an

3
n 3
2

1
m

n 1

n ( m 1)

m2 1
2 m
m 3
mn1

n 1

(China National Olympiad 2005)

eroberogeday lwm pln

- TMBr 3 -

KNitviTaCMuvijBiPBelak
*> eK[ a , b , c CacMnYnBitviCmanEdl a b c 3 .
a2
b2
c2
3

cUrRsaybBaak;vismPaB
2
2
2
2
ab

bc

ca

(Croatia Team Selection Tests 2011)

(>eK[ a , b , c CaRCugrbs;RtIekaNmYy . cUrRsayfa


( c a b ) 4 ( a b c )4 ( b c a ) 4

ab bc ca
a(a b c) b(b c a ) c(c a b )
(Greece National Olympiad 2007)

!0> eK[ f CaGnuKmn_kMNt;BI (0 , ) eTA IR ehIyepgpat;lkxN


f ( x ) f ( y ) f ( xy ) RKb; x 0 , y 0 nig f ' (1) .
cUrkMMNt;rkGnuKmn_ y f (x) .
!!> eKkMNt;cMnYn a0 , a1 , a2 , ...., an dUcxageRkam
1
ak 2
a 0 ; ak 1 ak
( n 1 , k 0 , 1 , 2 , ...., n 1 )
2
n
cUrbgajfa 1 n1 an 1 .
(IMO Longlists 1980)

!@> eK[ f CaGnuKmn_Cab; nig manedrIevelI IR EdlcMeBaHRKb; x, y IR


eKmanTMnak;TMng f (x y ) 1f (xf)(x)ff((yy)) nig f (x) 1 RKb; x .
cUrkMNt;GnuKmn_ y f (x) .
eroberogeday lwm pln

- TMBr 4 -

KNitviTaCMuvijBiPBelak
!#> cUrbgajfa
log a bc logb ca logc ab 4 (logab c log bc a log ca b )

cMeBaHRKb; a , b , c FMCagmYy .

( Malaysia National Olympiad 2010 )

!$> eK[rgVg;pit O kaM r nigRtIekaN ABC ERbRbYlcarwkeRkArgVg; .


bnat;kat;tam O kat;RCug AB nig AC erogKaRtg; M nig N .
kMNt;TItaMgcMNuc A nigGgt; [MN] edIm,I[pRkLaRtIekaN AMN
mantmtUcbMput ?
(RbLgsisBUEkRbcaMraCFanIPMeBj 2009)

!%> eKtag , , CargVas;mMukgRtIekaN ABC mYyEdlmanbrimaRt


2p nigkaMrgVg;carwkeRkA R .
a / cUrRsaybBaak;fa
2

R
2
2
2

cot cot cot 3 9 . 2 1

b/

etIeBlNaeTIbeK)ansmPaB ?

( Morocco National Olympiad 2011 )

eroberogeday lwm pln

- TMBr 5 -

KNitviTaCMuvijBiPBelak
3 1 1
1 0 0
!^> eK[m:aRTIs A 1 3 1 nig I 0 1 0
1 1 3
0 0 1
bgajfamansIVtcMnYnBitBIr (un ) nig ( vn ) Edlepgpat;
n IN : A n un . I v n .A

EdleKnwgbBaak;tYTUeTA

un

nig vn CaGnuKmn_n n .
!&> eK[ f CaGnuKmn_Cab; nig manedrIevelI IR EdlcMeBaHRKb; x, y IR
eKmanTMnak;TMng f ( x 2 y ) f (x)f (y ) nig f (x) 0 .
cUrkMNt;GnuKmn_ y f (x) .
!*> eK[ctuekaNe)a:g ABCD mYymanRCug AB a , BC b ,
CD c nig DA d .
P CacMnucmYyenAkgctuekaNehIy K , L , M , N CacMeNalEkg
ncMnuc P elIRCug [AB] , [BC] , [CD] , [DA] erogKa .
PL PM PN
eKdwgfa PK

h .
AB BC CD DA
k> cUrRsayfa h 2 22S 2 2 Edl S CapRkLan
a b c d
ctuekaN ABCD .
x>cUrbgajfa h 12
eroberogeday lwm pln

- TMBr 6 -

KNitviTaCMuvijBiPBelak
!(> eK[ctuekaNe)a:g ABCD mYymanRCug AB a , BC b

nig DA d . O CacMnucmYyenAkgctuekaNenHEdl
OAB OBC OCD ODA 0 90o )
k> cUrRsayfa tan a2 b24Sc2 d2
Edl S CapRkLanctuekaN ABCD .
x> kMnt; edaydwgfa tan mantmFMbMput .
CD c

@0> cMeBaHRKb; x ; y 0 cUrRsaybBaak;vismPaB


x 2 x 1 y 2 y 1 x 2 x 1 y 2 y 1 2( x y )

(Kazakhstan NMO 2010)

@!> eK[ a , b , c CacMnYnBitviCman . cUrRsaybBaak;fa


( 2a b c)2
( 2b c a)2
( 2c a b )2
2
2
8
2
2
2
2
2a (b c)
2b (c a)
2c (a b )
(USAMO 2003)

@@> cUrbgajfa

1
1 1

a b

1 1

c d
cMeBaHRKb;cMnYnBitviCman a , b , c , d

eroberogeday lwm pln

1
1

ac bd

evotNam

1962

- TMBr 7 -

KNitviTaCMuvijBiPBelak
@#> cUrbgajfa

1
1
1
4

( x y )2 ( y z )2 ( z x )2 xy yz zx

cMeBaHRKb;cMnYnBitminGviCmanxusKa x , y , z .

evotNam

2008

@$> cUrkMNt;tYTUeTAnsIVtEdlkMNt;eday
x 3, x 4 nig x x nx cMeBaHRKb; n .
7
@%> cUrkMNt;RKb;cMnYnBit x ebIeKdwgfa 128 27
.
18
6
@^> cUrKNnaplbUk
0

n 1

2
n 1

Sn

3
4
n2

...
1! 2! 3! 2! 3! 4!
n! (n 1)! (n 2)!

@&> ( China 1983 )


eK[ f CaGnuKmn_kMNt;elIcenaH [0 ,1] edaydwgfa
f (0) f (1) 1 nig | f (a) f (b) | | a b |
cMeBaHRKb; a b kgcenaH [0 ,1] .
cUrbgajfa | f (a) f (b) | 12 .
@*> ( AIME 1988 )
cUrkMNt;KUncMnYnKt; (a ,b) edaydwgfa ax bx 1
Eckdac;nwg x x 1 .
17

16

eroberogeday lwm pln

- TMBr 8 -

KNitviTaCMuvijBiPBelak
@(> edaHRsaykgsMNMucMnYnBitnsmIkar
x 3x x 2 .
#0> ( Korean Mathematics Competition 2000 )
cUrkMNt;RKb;cMnYnBit x Edlepgpat;smIkar
3

2x 3x 4x 6x 9x 1

#!> ( China 1992 )


cUrbgajfa 16

1
17
k 1
k
z1 z 2 z 2 2
#@> eK[ z1 , z 2 , z 3 CacMnYnkMupicEdl z12 z 22 z 32 3
z z z 4
1 2 3
cUrKNnatm S z z 1z 1 z z 1z 1 z z 1z 1
1 2
3
2 3
1
3 1
2

80

##> eK[ x , y , z CabIcMnYnBitviCmanEdl x y z 1 .


cUrkMNt;tmtUcbMputn 1 x x 1 y y 1 z z .
#$> ( Iran 1996 )
eK[bIcMnYnBitminGviCman a , b , c nigminsUnRBmKaBIr .
cUrRsaybBaak;fa
4

1
1
1 9
(ab bc ca)

2
2
2
(a b) (b c) (c a) 4

eroberogeday lwm pln

- TMBr 9 -

KNitviTaCMuvijBiPBelak
#%> eK[ a , b , c CabIcMnYnBitminGviCman .
cUrRsayfa a3 b3 c3 3abc 2 ( b 2 c a)3 .
#^> eK[RtIekaN ABC mYymanRCug BC a , CA b , AB c
ehIymMukg A , B , C CamMuRsYcbmMuEkg .
tag S CapRkLan ABC
cUrRsaybBaak;fa 14 14 14 9 2 .
a

16 S

#&> Turkey 2007


eK[bIcMnYnBitviCman a , b , c Edl a b c 1 .
cUrRsaybBaak;fa
1
1
1
1

ab 2c2 2c bc 2a2 2a ca 2b2 2b ab bc ca


#*> eK[ a , b , c CabIcMnYnBitviCman .

cUrRsaybBaak;fa
a2
2a2 ab b2

b2
2b2 bc c2

#(> eK[ a , b , c CabIcMnYnBitxusKa .


a2
b2
cUrRsayfa

2
2
(b c )

eroberogeday lwm pln

(c a )

c2
2c2 ca a2

c2
2
2
(a b )

abc
2

- TMBr 10 -

KNitviTaCMuvijBiPBelak
$0> cUrRsaybBaak;vismPaB
a 2 (1 b )2 b 2 (1 c)2 c2 (1 a )2

3 2
2

cMeBaHRKb;cMnYnBit a , b , c .
$!> eK[RtIekaN ABC mYyEkgRtg; C . D nig E CacMNucBIreRCIserIs
enAelIGIub:UetnUs Edl BC BD nig AC AE .
F nig G CacMeNalEkgn D nig E elIRCug AC nig BC erogKa .
cUrRsaybBaak;fa DE DF EG .
$@> eKyk D CacMnucmYynRCug BC rbs;RtIekaN ABC ehIy E
nig F CacMeNalEkgn B nig C elI AD .
ebI R CacMnuckNaln BC enaHRsayfa RE RF
$#> eK[kaer ABCD mYy . E CaRbsBVrvagGgt;RTUgTaMgBIr
ehIy N CacMnucmYysitelI AE .
cUrRsayfa AB2 BN 2 AN.NC nig AN 2 NC2 2BN 2
$$> eK[ ABC CaRtIekaNmYyehIy D CaeCIgnkm<s;KUsBIkMBUl A .
yk E nig F sitenAelIbnat;kat;tam D edaydwgfa AE Ekgnwg BE
ehIy AF Ekgnwg CF Edl E nig F xusBI D .
yk M nig N CacMNuckNalnGgt; BC nig EF erogKa .
cUrRsayfa AN Ekgnwg NM ?
eroberogeday lwm pln

- TMBr 11 -

KNitviTaCMuvijBiPBelak
$%> rkGnuKmn_ f : IR IR Edlepgpat;
x, y IR : f ((x y)2 ) x 2 2yf (x) f (y)

$^> cMeBaHRKb;cMnYnBitviCman a , b ,c cUrRsaybBaak;fa


2
2
2
2
2
2
(

)(

)(

) 3
a
b
b
c
c
a
3 (a b)(b c)(c a ) 6
abc
8

$&> Greece National Olympiad 2011


eK[ a , b , c CacMnYnBitviCmanEdlmanplbUkesI 6 .
cUrkMNt;tmGtibrman
S 3 a 2 2bc 3 b 2 2ca 3 c 2 2ab

$*>USAMO 1989
cMeBaHRKb;cMnYnKt;viCman n eK[
1 1
1
....
2 3
n
Tn S1 S 2 S 3 .... Sn

Sn 1

Un

T1 T2 T3
T

.... n
2
3
4
n1

cUrkMNt;edayeFIVdMeNaHRsay nUvcMnYnKt; 0 a, b, c, d 1000 000


edaydwgfa T1988 aS1989 b nig U1988 cS1989 d .

eroberogeday lwm pln

- TMBr 12 -

KNitviTaCMuvijBiPBelak
$(>(Japan Mathematical Olympiad Finals 2010)
eK[ x , y , z CacMnYnBitviCman . cUrbgajfa
1 yz zx 1 zx xy 1 xy yz

1
(1 x y)2 (1 y z)2 (1 z x)2

%0> China Team Selection Test 2002


eK[sIVt a 1 , a 5 nig a
1

n 1

ana n 1
an a
2

cUrkMNt;tYTUeTAnsIVt {a } CaGnuKmn_n n .
%!> China Team Selection Test 2006
eK[ x , x ,...., x CacMnYnBitviCmanEdl x
cUrRsayfa

1
n

.
x

2
n 1

n 2

i 1

i 1

1 .

i 1

1 xi

n1

%@> China Team Selection Test 2005


eK[ a , b ,c CabIcMnYnBitminGviCman Edl ab bc ca 13 .
cUrbgajfa a

1
1
1

3
2
2
2
bc 1 b ca 1 c ab 1

eroberogeday lwm pln

- TMBr 13 -

KNitviTaCMuvijBiPBelak

eroberogeday lwm pln

- TMBr 14 -

KNitviTaCMuvijBiPBelak
CMBUkTI2

EpkdMeNaHRsay
lMhat;TI1
eK[cMnYnBit x1 , x2 , y1 , y 2 , z1 , z 2 epgpat; x1 0 , x2 0
x1y1 z12 0 nig x 2y 2 z 2 2 0 .
cUrRsayfa
8
1
1

( x1 x 2 )( y1 y 2 ) ( z1 z 2 )2 x1y1 z12 x 2y 2 z 2 2

( IMO 1969 )

dMeNaHRsay
Rsayfa
8
(x1 x2 )(y1 y2 ) (z1 z2 )

x1y1 z1

1
x2y2 z2

(*)

tag a x1y1 z12 0 ; b x2y 2 z 22 0


nig c (x1 x2 )(y1 y 2 ) (z1 z 2 )2
a z 12
eday x1 0 nig x2 0 enaH y1 x 0 nig y 2 0
1

eroberogeday lwm pln

- TMBr 15 -

KNitviTaCMuvijBiPBelak
eKman
c ( x1y1 z12 ) ( x 2y 2 z 2 2 ) x1y 2 x 2y1 2z1z 2
c a b x1y 2 x 2y1 2z1z 2

eday a x1y1 z1 enaH

a z 12
x1
y1

nig

b z 22
x2
y2

b x2y 2 z 2

enaH

a z 12
b z 22
eK)an c a b y 2 ( y ) y1 ( y ) 2z1z 2
1
2
y
y
y
y
a b 2 a 1 b 2 z 12 2 z 1z 2 1 z 2 2
y1
y2
y1
y2
y

y
y
y1
a b 2 a 1 b 2 z1
z 2
y1
y2
y2
y1

eday yy2 z1 yy1 z 2 0 enaHeKTaj)an


2
1

y
y
y
y
c a b 2 a 1 b eday 2 a 1 b 2 ab
y1
y2
y1
y2
enaH c a b 2 ab ( a b )2 (1)
eyIg]bmafavismPaB (*) BiteBalKW 8c a1 b1 Bit

eK)an c a8abb eday c (


eroberogeday lwm pln

a b )2

tamvismPaB (1)
- TMBr 16 -

KNitviTaCMuvijBiPBelak
eyIgnwgRsayfa (
smmUl (a b)(

8ab
ab
a b )2 8ab
a b )2

tamvismPaB AM GM eKman
a b 2 ab ehIy ( a b )2 4
enaH (a b)( a b ) 8ab Bit
dUcenH

ab

8
1
1

( x1 x2 )(y1 y 2 ) ( z1 z 2 )2 x1y1 z12 x2y 2 z 22

vismPaBenHkaysmPaBluHRtaEt
y2
y
y
y
z1
z 2 1 b z1y 2 z 2y1 nig 2 a 1 b .
y
y
y
y
1

eroberogeday lwm pln

- TMBr 17 -

KNitviTaCMuvijBiPBelak
lMhat;TI2
eKtag I nig O erogKaCapitrgVg;carwkkgnigpitrgVg;carwkeRkA
nRtIekaN eK[RtIekaN ABC mYy .
cUrRsayfa OIA 90o luHRtaEt AB , BC , CA CasIVtnBVn .
dMeNaHRsay
Rsayfa OIA 90o luHRtaEt AB , BC , CA CasIVtnBVn
C

I
O
A

tag BC a , AC b , AB c nig p a b2 c
ehIy r nig R CakaMrgVg;carwkkg nig carwkeRkAnRtIekaN ABC .
yk H CacMeNaln I elI [AC] enaHeK)an
IH r nig AH p a .
eroberogeday lwm pln

- TMBr 18 -

KNitviTaCMuvijBiPBelak
-sntfa OIA 90o enaH OA 2 OI 2 IA 2
tamRTwsIbTGWEleKman OI 2 R(R 2r )
tamRTwsIbTBItaKrkgRtIekaNEkg AHI : IA 2 AH 2 IH 2
eK)an R 2 R(R 2r ) r 2 (p a)2
b 2rR r 2 (p a)2
tamrUbmnehr:ug S pr p(p a)(p b)(p c) abc
4R
eKTaj
eK)an

abc
(p a )(p b )(p c)
ni
g
r2
2p
p
abc (p a )(p b )(p c)

(p a )2
2p
p
2rR

abc 2(p a )(p b )(p c) 2p(p a )2


abc 2(p a )[(p b )(p c) p(p a )]
abc ( 2p 2a )(p 2 pb pc bc p 2 pa )
abc ( 2p 2a )[2p 2 p(a b c) bc]
abc (b c a )( 2p 2 2p 2 bc )
abc bc(b c a )
abca
eKTaj 2a b c enaH c , a , b

CasIVtnBVn .
-sntfa c , a , b CasIVtnBVnenaHeK)an 2a b c
tamRTwsIbTkUsIunUskgRtIekaN OIA eK)an
eroberogeday lwm pln

- TMBr 19 -

KNitviTaCMuvijBiPBelak
OA 2 OI 2 IA 2 2OI .IA cos OIA
OI 2 IA 2 OA 2
eKTaj cos OIA 2OI.IA
R( R 2r ) r 2 (p a )2 R 2

2OI .IA

r 2 2rR (p a )2

2OI .IA
a b c 3a
eday b c 2a enaH p 2 2
3a
3a
3a
( a )( b )( c) a( 3a 2b )( 3a 2c)
2
2
r2 2

3a
12a
2
9a 2 6(b c)a 4bc 4bc 3a 2

12
12
bc

ehIy R 2rR abc


3a
2( ) 3
2
4bc 3a 2 bc 3a
( a )2
0
eK)an cos OIA 12 2OI3.IA 2
eK)an OIA 90o .

dUcenH

OIA 90o

eroberogeday lwm pln

luHRtaEt

AB , BC , CA

CasIVtnBVn .
- TMBr 20 -

KNitviTaCMuvijBiPBelak
lMhat;TI3
eK[ n CacMnYnKt;viCmanedaydwgfa 2 2 28n2 1 CacMnYnKt;.
cUrRsayfa 2 2 28n2 1 CakaerR)akdncMnYnKt;mYy .
dMeNaHRsay
rebobTI1
Rsayfa 2 2 28n2 1 CakaerR)akdncMnYnKt;
tambRmab; 2 2 28n2 1 CacMnYnKt;naM[man m IN Edl
28n 2 1 m 2 b m 2 28n 2 1 CasmIkar Pell .
KUcemIydMbUgnsmIkarenHKW m 127 , n 24
eRBaH 1272 28 242 1 . cMeBaHRKb; k 1eKGacsresr
m 2 28n 2 127 2 28 242 (127 28 242 )k
(m 2 7n )(m 2 7n ) (127 48 7 )k (127 48 7 )k
m 2 7n (127 48 7 )k
eKTaj
m 2 7n (127 48 7 )k

eK)ancemIyTUeTAnsmIkar
(127 48 7 )k (127 48 7 )k
m
2
(127 48 7 )k (127 48 7 )k
n
4 7

eroberogeday lwm pln

- TMBr 21 -

KNitviTaCMuvijBiPBelak
kgkrNIenHeK)an

2 2 28n 2 1 2 2m

2 2 28n 2 1 2 (127 48 7 )k (127 48 7 )k

eday 127 48 7 (8 3 7 )2
nig (8 3 7 )(8 3 7 ) 1 enaHeK)an

k 2

2 (127 48 7 ) (127 48 7 ) (8 3 7 ) (8 3 7 )
k

eK)an 2 2 28n2 1 [(8 3 7 )k (8 3 7 )k ]2


CakaerR)akdncMnYnKt;eRBaH (8 3 7 )k (8 3 7 )k CacMnYnKt;
dUcenHebI 2 2 28n2 1 CacMnYnKt;enaH 2 2 28n2 1
CakaerR)akdncMnYnKt; .
rebobTI2
Rsayfa 2 2 28n2 1 CakaerR)akdncMnYnKt;
tambRmab; 2 2 28n2 1 CacMnYnKt;naM[man m IN Edl
28n 2 1 m 2 b m 2 1 28n 2
b m2 1 . m2 1 7n2
tamsmIkarenHeKTaj)an

m 1
2
p

m 1 7q 2
2

Edl p nig q CacMnYnKt;viCman .


eroberogeday lwm pln

m 1
2
7
p

m 1 q2
2

- TMBr 22 -

KNitviTaCMuvijBiPBelak
eKTaj)an
m 2p 2 1 , m 14q 2 1 b m 14p 2 1 , m 2q 2 1

-krNI m 2p2 1 , m 14q2 1


eK)an 2p2 1 14q2 1 b p2 7q2 1 CasmIkarKan
cemIykg IN eRBaHGgTIBIrsmIkarEcknwg 7 [sMNl; 1
EtGgTIBIrnsmIkarEcknwg 7 minGac[sMNl; 1 eT eRBaHRKb;
cMnYnKt;viCman p cMnYn p2 Ecknwg 7 [sMNl; 1 , 2 , 4 .
-krNI m 14p2 1 , m 2q2 1
eK)an 14p2 1 2q2 1 b q2 7p2 1 CasmIkarmancemIy
kgsMNMu IN eRBaH q2 7p2 Ecknwg 7 Gac[sMNl; 1 .
ehtuenH manKU p , q IN Edl m 14p2 1 , m 2q2 1
kgkrNIenHeK)an 2 2 28n2 1 2 2m
cMeBaH m 2q2 1 eK)an
2 2 28n 2 1 2 2( 2q 2 1) 4q 2 CakaerR)akd .
beKGacyk m 14p2 1 eK)an
2 2 28n 2 1 2 2(14p 2 1) 28p 2 4

CakaerR)akd
b 7p2 1 q2 .

4(7p 2 1) 4q 2

eRBaH q2 7p2 1
eroberogeday lwm pln

- TMBr 23 -

KNitviTaCMuvijBiPBelak
rebobTI3 Rsayfa 2 2 28n2 1 CakaerR)akdncMnYnKt;
tambRmab; 2 2 28n2 1 CacMnYnKt;naM[mancMnYnKt;viCman p
Edl 28n2 1 (2p 1)2 4p2 4p 1
eK)an p(p 1) 7n2 eday GCD(p , p 1) 1
eRBaH (p 1) p 1 tamRTwsIbT Bezout
eKTaj)an p Eckdac;nwg 7 b p 1 Eckdac;nwg 7 .
-krNI p Eckdac;nwg 7
tam p(p 1) 7n2 eKTaj)an p 7k 2 , p 1 t 2
RKb;cMnYnKt;viCman k nig t ehIy GCD(k , t ) 1 .
eKman 7k 2 1 t 2 b t 2 7k 2 1 CasmIkarmanbskg IN
eRBaH t 2 7k 2 Ecknwg 7 mansMNl; 1 , 2 b 4 .
-krNI p 1 Eckdac;nwg 7
tam p(p 1) 7n2 eKTaj)an p k 2 , p 1 7t 2
RKb;cMnYnKt;viCman k nig t ehIy GCD(k , t ) 1 .
eKman k 2 1 7t 2 b k 2 7t 2 1 CasmIkarKanbskg IN
eRBaH k 2 7t 2 Ecknwg 7 minGacmansMNl; 1 eT .
cMeBaH p 7k 2 , p 1 t 2 Edl t 2 7k 2 1 eKman
2 2 28n 2 1 2 2( 2p 1) 4p 4 4t 2 Cakaer .
eroberogeday lwm pln

- TMBr 24 -

KNitviTaCMuvijBiPBelak
lMhat;TI4
eKyk x CacMnYnBit edaydwgfa 0 x 2 .
cUrbgajfa cos2 (x) cot(x) sin 2 (x) tan( x) 1
( Baltic Way 2010 )

dMeNaHRsay
bgajfa cos2 (x) cot(x) sin 2 (x) tan( x) 1
x
sin x
tan x
edayCMnYs cot x cos
ni
g
vi
s
mPaBsmmU
l
sin x
cos x
cos 3 x sin 3 x

1 ehIy sin x 0 , cos x 0 x (0, )


sin x
cos x
2
tamvismPaB AM GM eK)an
cos 3 x cos 3 x

sin 2 x 3 cos2 x
sin x
sin x
cos 3 x 3 cos 2 x sin 2 x
b sin x
(1)
2
sin 3 x sin 3 x
ehIy cos x cos x cos2 x 3 sin 2 x
sin 3 x 3 sin 2 x cos 2 x
b cos x
( 2)
2

eroberogeday lwm pln

- TMBr 25 -

KNitviTaCMuvijBiPBelak
bUkvismPaB (1) nig (2) GgnigGgeK)an
cos 3 x sin 3 x

cos 2 x sin 2 x 1 Bit


sin x
cos x
dUcenH cos2 (x) cot(x) sin 2 (x) tan( x) 1 .

eroberogeday lwm pln

- TMBr 26 -

KNitviTaCMuvijBiPBelak
lMhat;TI5
eK[RtIekaN ABC mYymanRCug a , b , c . tag r nig R erogKaCa
kaMrgVg;carwkkg nig kaMrgVg;carwkeRkAn ABC .
k> cURsayfa a cos A bosB c cos C 2Rpr
cos A cos B cos C a 2 b 2 c 2

a
b
c
2abc
r
cos A cos B cos C 1
R
abc
p
2
a 2 b 2 c 2 4 ( R r )2 A , B, C

Edl
CaknHbrimaRtnRtIekaN .
x> cUrTajbBaak;fa

dMeNaHRsay
k> Rsayfa a cos A bosB c cos C 2Rpr

CamMuRsYc.

M
O

eroberogeday lwm pln

- TMBr 27 -

KNitviTaCMuvijBiPBelak
eKman BAC 12 BOC BOK mMupit nig mMucarwkkgrgVg;
OK
kgRtIekaNEkg OKB eKman cos BOK OK

R
OB
eKTaj OK R cos BOK R cos A .
RsaydUcKaEdr OL R cos B , OM R cos C
tag S CapRkLanRtIekaN ABC enaHeK)an
S SOBC SOCA SOAB
1
1
1
pr BC.OK CA.OL AB.OM
2
2
2
1
1
1
pr a R cos A bR cos B cR cos C
2
2
2
1
pr R(a cos A b cos B c cos C)
2
2pr
a cos A b cos B c cos C
R
a 2 b 2 c 2 2bc cos A

dUcenH
tamRTwsIbTkUsIunUseKman
c a
eKTaj cosa A b 2abc
.
RsaydUcKaEdreK)an
cos B a c b
cos C a

ni
g
b
2abc
c
2

b 2 c2
2abc
cos A cos B cos C a 2 b 2 c 2

a
b
c
2abc
2

dUcenH

eroberogeday lwm pln

.
- TMBr 28 -

KNitviTaCMuvijBiPBelak
ma:geToteKman
b 2 c 2 a2 c 2 a 2 b 2 a 2 b 2 c 2

cos A cosB cosC


2bc
2ac
2ab
a(b2 c2 ) b(c2 a2 ) c(a2 b2 ) (a3 b3 c3 )

2abc
(a b c)(a2 b2 c2 ) 2(a3 b3 c3 )

(*)
2abc

tamrUbmnehr:ugeKman p(p a)(p b)(p c) pr


elIkGgTaMgBIrCakaereK)an
p(p a )(p b )(p c) p 2r 2
(p a )(p b )(p c) pr 2

p 3 (a b c)p 2 (ab bc ca)p abc pr 2


abc
a b c 2p
abc 4Rpr
S
pr
4R
p 3 2p 3 (ab bc ca)p 4Rpr pr 2

eday
ehIy
enaH
eK)an
eKTaj ab bc ca p r 4rR
eday (a b c) a b c 2(ab bc ca)
eK)an a b c 4p 2(p r 4rR )
b a b c 2(p r 4rR )
ehIy a b c 3abc (a b c)(a b c ab bc ca)
eKTaj)an a b c 2(p 3pr 6Rpr )
2

eroberogeday lwm pln

- TMBr 29 -

KNitviTaCMuvijBiPBelak
TMnak;TMng (*) Gacsresr
4p(p 2 r 2 4Rr ) (p 3 3pr 2 6Rpr )
cos A cos B cos C
8Rpr
p 2 r 2 4Rr p 2 3r 2 6Rr

2Rr
2r 2 2Rr r

1
2Rr
R

dUcenH cos A cos B cos C 1 Rr .


x> TajbBaak;fa a b c 4 (R r )
tamvismPaB Cauchy Schwarz eKman
2

( x1y1 x 2 y 2 x 3 y 3 )2 ( x1 x 2 x 3 )( y 1 y 2 y 3 )
2

yk x
nig y

a cos A , x 2 b cos B , x 3 c cos C

cos C
cos B
cos A

,
z
,
y
1
2
2
c
b
a
2
2
2pr a b c 2
2
(cos A cos B cos C)
.
R
2abc
r 2 2pr a 2 b 2 c 2
(1 )
.
R
R
8Rpr

dUcenH

eK)an

(r R ) 2 a 2 b 2 c 2

2
R
4R 2
a 2 b 2 c 2 4 ( R r )2

eroberogeday lwm pln

- TMBr 30 -

KNitviTaCMuvijBiPBelak
lMhat;TI6
eK[ f CaGnuKmn_kMNt;BIsMNMu IR eTAsMNMu IR* Edlepgpat;
lkxN f (x y ) f (x).f (y ) cMeBaHRKb; x , y IR nig f '(0)
cUrkMNt;rkGnuKmn_ y f (x) .
dMeNaHRsay
kMNt;rkGnuKmn_ y f (x)
eKman f (x y ) f (x).f (y ) cMeBaHRKb; x , y IR
yk x y 0 eK)an f (0) f (0)
naM[ f (0)[1 f (0)] 0 eKTaj f (0) 0 b f (0) 1
eday f CaGnuKmn_kMNt;BIsMNMu IR eTAsMNMu IR* enaH f (0) 0
eK)an f (0) 1 .
eFIVedrIeveFobnwg x kgsmPaB f (x y ) f (x).f (y ) edaycat;Tuk y
CaGefrminGaRsynwg x eK)an f '(x y ) f '(x)f (y )
yk x 0 eK)an f '(y ) f '(0)f (y ) eday f '(0)
eKTaj)an f '(y ) f (y ) 0 CasmIkarDIepr:g;EsllIenEGlMdab;TI1
eK)ancemIyTUeTAnsmIkarrag f (y ) k e
cMeBaH y 0 eK)an f (0) k 1 eRBaH f (0) 1
dUcenH f (x) e CaGnuKmn_RtUvrk .

eroberogeday lwm pln

- TMBr 31 -

KNitviTaCMuvijBiPBelak
lMhat;TI7
21
sIVt {an } kMNt;eday a1 16
nigcMeBaH n 2 : 2an 3an1 n31
2
eKyk m CacMnYnKt;mYyEdl m 2 . cUrbgajfacMeBaH n m
eyIg)an an

2n 3

n ( m 1)

m2 1
2 m
m 3
mn1

1
m

(China National Olympiad 2005)

dMeNaHRsay
bgajfa

an n 3
2

eKman 2an 3an 1

1
m

n ( m 1)

m2 1
2 m
mn1
m 3

3
2n 1

naM[ 2n an 3.2n 1 an 1 43
n

n 1

b 23 an 23 an 1 43 1n
3


k 1
n 2 k
3 n 1

2
eK)an 3 ak 3 ak 1 4 k

k 2 3
k 2

eroberogeday lwm pln

- TMBr 32 -

KNitviTaCMuvijBiPBelak
n 1

1
1


n
2
3 1
2
3
an a1 2 .
1
3
4 3
3
1
3
n
2 21 1
1
2
an . 1 n 1
3 16 8
3
3
n
n
3
3
3
3
eKTaj)an an 2 n 3 b an n 3 2


2
2
1
n ( m 1)
tag P an n3 3 m m 23 m
2

n
n ( m 1)
3 m
2 m
eK)an P 2 m 3

m2 1
m 1
1)
eKman m n 1 (m(m 1)(1m

n
)n
1
m1
m2 1
edIm,IRsay[eXIjfa P m n 1
n
eyIgRtUvRsay[eXIjfa (1 m 1) P m 1
tamvismPaB Bernoully eKman

eroberogeday lwm pln

- TMBr 33 -

KNitviTaCMuvijBiPBelak

n
n
1 n m
1

(1
1
)

m1
m1
m 1
1 1

m
mn

n
1
1

1
naM[ m 1

1 m

1 1
(1 )

m
m
cMeBaHRKb; m 2 tameTVFajtun eKman
1

enaH 1 m

1 2
1
1 1
Cm 2 Cm
.... m Cm
m
m
m
m

1 2 5 1
9 3
1
1 C1m 2 Cm


2 2m 4 2
m
m

eKTaj)an 1 mn 1

2

3

2n
2m

eKTaj 1 mn 1 P 3

2n

2n
2 m

n

m 1 3

n ( m 1)

Et P 2 m 23 m

n
n
( m 1)

n
2 m
2 m
eK)an (1 m 1) P 3 m 3

n
3 m

eroberogeday lwm pln

- TMBr 34 -

KNitviTaCMuvijBiPBelak
n
2 m

]bmafa 3

( m 1)

2 m

m
m1

Bit

n
2 m

edaytag u 3 enaH u(m um 1 ) m 1



smmUl mu um m 1 0
m(u 1) (um 1) 0
(u 1)[m (um 1 .... u 1)] 0
n
2 m

eday m n & m 2 enaH 0 u 3 1



naM[ (u 1)[m (um 1 .... u 1)] 0 Bit
dUcenH a 3 m 2 m 1
n ( m 1)
m

1
m

2n 3

eroberogeday lwm pln

mn1

- TMBr 35 -

KNitviTaCMuvijBiPBelak
lMhat;TI8
eK[ a , b , c CacMnYnBitviCmanEdl a b c 3 .
a2
b2
c2
3
cUrRsaybBaak;vismPaB

2
2
2
2
ab

bc

ca

(Croatia Team Selection Tests 2011)

dMeNaHRsay
a2
b2
c2
3
RsaybBaak;vismPaB

a b 2 b c2 c a2 2
a2
a(a b 2 ) ab 2
ab 2
a
(1)
eKman

2
2
2
ab
ab
ab
b2
bc 2
c2
ca2
dUcKaEdr 2 b 2 (2) ; 2 c 2 (3)
bc
bc
ca
ca
a2
b2
c2
tag S 2 2 2 . bUkvismPaB (1), (2) & (3)
ab
bc
ca
ab 2
bc 2
ca2
eK)an S 3 ( 2 2 2 )
ab
bc
ca
edIm,IRsayfa S 32 enaHeyIgnwgRsayfa
ab 2
bc 2
ca2
3 3
3
S
3

.
2
2
2
2
2
ab
bc
ca
2ab 2
2
tamvismPaB AM GM eKman a b 2b a b a

eroberogeday lwm pln

- TMBr 36 -

KNitviTaCMuvijBiPBelak
ab 2
b a
bc 2
c b ca2
a c
,
eKTaj

ehI
y

2
2 c a2
2
a b2
b c2
ab 2
bc 2
ca2
1
ehtuenH

(a c b a c b ) (*)
2
2
2
2
ab
bc
ca
tamvismPaB Cauchy Schwarz eKman
(a c b a c b )2 (a b c)(ab bc ca)
(a c b a c b )2 3(ab bc ca )

ehIy (ab bc ca)2 (a2 b2 c2 )(b2 c2 a2 )


(ab bc ca)2 (a 2 b 2 c 2 )2
ab bc ca a 2 b 2 c2
ab bc ca (a b c)2 2(ab bc ca)
3(ab bc ca) (a b c)2 9

eKTaj (a c b a c b )2 3(ab bc ca) 9


naM[ a c b a c b 3 (**)
tamvismPaB (*) & (**) eKTaj)an
ab 2
bc 2
ca2
3

Bi
t
2
2
2
2
ab

dUcenH

bc
ca
a2
b2
c2
3

a b 2 b c2 c a2 2

eroberogeday lwm pln

- TMBr 37 -

KNitviTaCMuvijBiPBelak
lMhat;TI9
eK[ a , b , c CaRCugrbs;RtIekaNmYy . cUrRsayfa
( c a b ) 4 ( a b c )4 ( b c a ) 4
ab bc ca

a(a b c) b(b c a ) c(c a b )

dMeNaHRsay
Rsayfa

(Greece National Olympiad 2007)

( c a b ) 4 ( a b c )4 ( b c a ) 4

ab bc ca
a(a b c) b(b c a ) c(c a b )

tamvismPaB AM GM eKman
( c a b )4
a(a b c) 2(c a b )2
a( a b c )

eKTaj

( c a b )4
2(c a b )2 a(a b c)
a( a b c )

( c a b )4
a 2 2b 2 2c 2 5ac 5ab 4bc (1)
a( a b c )

b
RsaydUcKaEdreK)an

( a b c )4
2a 2 b 2 2c 2 5ab 5bc 4ac ( 2)
b(b c a )
( b c a )4
2a 2 2b 2 c 2 5bc 5ac 4ab ( 3)
c(c a b )

eroberogeday lwm pln

- TMBr 38 -

KNitviTaCMuvijBiPBelak
bUkvismPaB (1) ,

( 2) & ( 3)

eK)an

S 5(a 2 b 2 c 2 ) 4(ab bc ca) ab bc ac

dUcenH

Bit

(c a b)4 (a b c)4 (b c a)4


S

ab bc ca
a(a b c) b(b c a) c(c a b)

eroberogeday lwm pln

- TMBr 39 -

KNitviTaCMuvijBiPBelak
lMhat;TI10
eK[ f CaGnuKmn_kMNt;BI (0 , ) eTA IR ehIyepgpat;lkxN
f ( x ) f ( y ) f ( xy ) RKb; x 0 , y 0 nig f ' (1) .
cUrkMMNt;rkGnuKmn_ y f (x) .
dMeNaHRsay
kMMNt;rkGnuKmn_ y f (x)
eKman f (x) f (y ) f (xy) RKb; x 0 , y 0
yk y 0 eK)an f (x) f (1) f (x) naM[ f (1) 0
eFIVedrIeveFobnwg x kgsmPaB f (x) f (y ) f (xy) edaycat;Tuk y
CaGefrminGaRsynwg x eK)an f '(x) y f '(xy)
yk x 1 eK)an f '(1) yf '(y ) eday f '(1) enaH f '(y ) y
eKTaj)an f (y ) y dy ln | y | k
yk y 1 enaH f (1) k 0 eRBaH f (1) 0 .
ehtuenH f (y ) ln | y | ln y RKb; y 0
dUcenH f (x) . ln x CaGnuKmn_RtUvrk .

eroberogeday lwm pln

- TMBr 40 -

KNitviTaCMuvijBiPBelak
lMhat;TI11
eKkMNt;cMnYn a0 , a1 , a2 , ...., an dUcxageRkam
1
ak 2
a 0 ; ak 1 ak
( n 1 , k 0 , 1 , 2 , ...., n 1 )
n
2
cUrbgajfa 1 n1 an 1 .
(IMO Longlists 1980)

dMeNaHRsay
bgajfa 1 n1 an 1
ak 2 a k (n a k )
eKman ak 1 ak n n
eK)an a 1 a (an n) a1 a 1 n
k 1
k k
k
k
b a 1 n a1 a 1
k
k
k 1
n 1
n 1 1
1
ehtuenH a n ( a a 1 ) a1 a1 2 a1 (*)
k 0 k
k 0 k
k 1
0
n
n
eKman a0 12 0 Bit . ]bmafa ak 0 Bit
ak 2
tam ak 1 ak n eKTaj)an ak 1 0 Bit

eroberogeday lwm pln

- TMBr 41 -

KNitviTaCMuvijBiPBelak
1
1
, n 1
k n n
1 1 1
1 n
... 1
n
n
n
n n

dUcenH ak 0 enaH ak n n b a
eK)an

n 1

k 0

n 1
1

ak n k 0

(n )

tam (*) eKTaj)an 2 a1

naM[ an 1

(i )

ma:geToteday an 1 enaH ak 1 b ak n n 1
b a 1 n n 1 1 RKb; k 0 , 1 , 2 , ...., n 1 .
k

n 1

n 1

1
n

k 0
k 0 n 1 n 1
edayBiniteXIjfaRKb; n 1 eKman n n 1 nn 12 22 0
n 1
n 1
enaHeKTaj)an a 1 n n n 1 nn 12
k 0 k
tam (*) eKTaj)an 2 a1 nn 12
n
b a1 2 nn 12 n n 1 enaH an n n 1 1 n1 (ii )
n
tam (i ) & (ii ) eKTaj)an 1 n1 an 1 .
dUcenH 1 n1 an 1 .

eK)an

ak n

eroberogeday lwm pln

- TMBr 42 -

KNitviTaCMuvijBiPBelak
lMhat;TI12
eK[ f CaGnuKmn_Cab; nig manedrIevelI IR EdlcMeBaHRKb; x, y IR
eKmanTMnak;TMng f (x y ) 1f (xf)(x)ff((yy)) nig f (x) 1 RKb; x .
cUrkMNt;GnuKmn_ y f (x) .
dMeNaHRsay
kMNt;GnuKmn_ y f (x)
eKman f (x y ) 1f (xf)(x)ff((yy))
yk y 0 eK)an f (x) 1f (xf)(x)ff((00))
b f (x) f (x)f (0) f (x) f (0)
b f (0)[f (x) 1] 0 eKTaj)an f (0) 0 b f (x) 1
edaytamsmtikm f (x) 1 enaH f (0) 0 .
eFIVedrIeveFobnwg x kgsmPaB f (x y ) 1f (xf)(x)ff((yy))
edaycat;Tuk y CaGefrminGaRsynwg x eK)an
2

f ' ( x )[1 f ( x )f ( y )] f ' ( x )f ( y )[f ( x ) f ( y )]


[1 f ( x )f ( y )]2
f ' ( x )[1 f 2 ( y )]
f '(x y )
[1 f ( x )f ( y )]2

f '(x y )

eroberogeday lwm pln

- TMBr 43 -

KNitviTaCMuvijBiPBelak
f ' (0)[1 f 2 ( y )]
f '(y )
[1 f (0)f ( y )]2

yk x 0 eK)an
eday f (0) 0
eK)an f '(y ) f '(0)[1 f (y )] tag f '(0)
eK)an 1 f 'f(2y()y ) eRBaH f (y ) 1
eFIVGaMgetRkaleFobnwg y elIGgTaMgBIrnsmIkarxagelIeK)an
2

f ' ( y )dy
dy y k

1 f 2 (y )

tag z f (y ) enaH dz f '(y )dy


dz
1
1
1
(
eK)an 1f '(yf 2)dy

)dz

2
2 1 z 1 z
(y )
1 z
1 dz
1 dz

2 1 z 2 1 z
1
1
ln | 1 z | ln | 1 z |
2
2
1 1 z 1 1 f (y )
ln
ln
2 1 z 2 1 f (y )

eKTaj 12 ln 11 ff ((yy )) y k
cMeBaH y 0 eK)an 12 ln 11 ff ((00)) k naM[ k 0 eRBaH f (0) 0
ehtuenH 12 ln 11 ff ((yy )) y
eroberogeday lwm pln

- TMBr 44 -

KNitviTaCMuvijBiPBelak
eKTaj)an 11 ff ((yy )) e2 y (1) b 11 ff ((yy )) e 2 y (2)
tam (1) eK)an 1 f (y ) e f (y )e
1 e2 y
naM[ f (y ) 1 e2 y .
tam (2) eK)an 1 f (y ) e f (y )e
1e 2 y
naM[ f (y ) 1 e 2 y .

2y

2y

2 y

2y

dUcenH

1 e 2x
f ( x)
1 e 2 x

eroberogeday lwm pln

1 e 2x
f ( x)
1 e 2 x

Edl f '(0) .

- TMBr 45 -

KNitviTaCMuvijBiPBelak
lMhat;TI13
cUrbgajfa
log a bc logb ca logc ab 4 (logab c log bc a log ca b )

cMeBaHRKb; a , b , c FMCagmYy .

( Malaysia National Olympiad 2010 )

dMeNaHRsay
karbgaj
tag A loga bc logb ca logc ab
nig B 4 (logab c logbc a logca b)
ehIyyk x ln a , y ln b , z ln c
cMeBaH a , b , c 1 enaH x , y , z 0
p
eK)an

tamrUbmnbreKal logq p ln
ln q
yz zx xy
1 1
1 1
1 1
x( ) y ( ) z ( )

x
y
z
y z
z x
x y
z
x
y
4x
4y
4z
B 4(

xy yz zx yz zx xy
A

eroberogeday lwm pln

- TMBr 46 -

KNitviTaCMuvijBiPBelak
tamvismPaB AB GM RKb; u , v 0 : u v 2 uv
4
1 1
4
v

b (u v )2 4uv b uuv

b
u v u v u v
eKTaj x( y1 1z ) y4xz ; y( 1z 1x ) z4yx ; z( 1x y1 ) x4zy
bUkvismPaBenHeK)an A B Bit .
dUcenH
log a bc log b ca log c ab 4 (logab c log bc a log ca b )

eroberogeday lwm pln

- TMBr 47 -

KNitviTaCMuvijBiPBelak
lMhat;TI14
eK[rgVg;pit O kaM r nigRtIekaN ABC ERbRbYlcarwkeRkArgVg; .
bnat;kat;tam O kat;RCug AB nig AC erogKaRtg; M nig N .
kMNt;TItaMgcMNuc A nigGgt; [MN] edIm,I[pRkLaRtIekaN AMN
mantmtUcbMput ?
(RbLgsisBUEkRbcaMraCFanIPMeBj 2009)

dMeNaHRsay
kMNt;TItaMgcMNuc A nigGgt; [MN]
tag S AMN CaRkLapnRtIekaN
AMN ehIy A' , B' , C'
CacMeNalEkgn O elI
RCug BC , CA , AB
erogKa .
eyIg)an
S AMN

1
1
AM .AN. sin A r ( AM AN )
2
2
AM .AN. sin A r ( AM AN )

eKTaj)an
tamvismPaB AM GM eKman AM AN 2
eroberogeday lwm pln

AM .AN

- TMBr 48 -

KNitviTaCMuvijBiPBelak
eK)an AM.AN. sin A 2r AM.AN
b AM 2 .AN2 . sin2 A 4r 2 .AM.AN
4r 2
eKTaj)an AM.AN. sin A sin A
4r 2
ehtuenH S AMN sin A
edIm,I[pRkLaRtIekaN AMN mantmtUcbMputluHRtaEt sin A 1

eKTaj)an A 90o .
kgkrNIenHeFIV[vismPaB AM AN 2 AM.AN kayCasmPaB
KW AM AN ehtuenH AMN KWCaRtIekaNsm)atkMBUl .
eday OA CaknHbnat;BuHkgnmMu BAC enaH OA MN
ehtuenH OM ON naM[ MN 2. OM
OC'
r
kgRtIekaNEkg OMC' eKman sin OMC' OM

OM
eday OMC' 45o enaH OM 2 r
eKTaj)an MN 2 2 r .
snidan
edaymMu MAN 90o enaHcMnuc M KWCaknHrgVg;manpit O nigman
viCmaRt MN 2 2 r .
eroberogeday lwm pln

- TMBr 49 -

KNitviTaCMuvijBiPBelak
lMhat;TI15
eKtag , , CargVas;mMukgRtIekaN ABC mYyEdlmanbrimaRt 2p
nigkaMrgVg;carwkeRkA R .
R2

a / cUrRsaybBaak;fa cot cot cot 3 9 . 2 1


p

b / etIeBlNaeTIbeK)ansmPaB ?
2

( Morocco National Olympiad 2011 )

dMeNaHRsay
R2

a / RsaybBaak;fa cot cot cot 3 9 . 2 1


p

eKman

1
1
1
2
2
2

cot

cot

cot

2
2
2
sin sin sin
1
1
1
27 R 2
eyIgnwgRsayfa 2 2 2 2
sin sin sin
p

tamvismPaB Cauchy Schwarz eKman


1
1
1
1 1
1
1 2

(
) (1)
2
2
2

3
sin
sin
sin
sin sin sin

edayeRbI

2
(a 1 a 2 a 3 ) 2
a12 a 2 2 a 3

b1 b 2 b 3
b1 b 2 b 3

eroberogeday lwm pln

- TMBr 50 -

KNitviTaCMuvijBiPBelak
eK)an

1
1
1
9

sin sin sin sin sin sin

tamRTwsIbTsIunUsGnuvtn_kg ABC eK)an

a
b
c
abc
2R

sin sin sin sin sin sin


eKTaj sin sin sin a b c p
2R
R
9
9R
ehtuenH 1 1 1

( 2)
sin sin sin sin sin sin
p

tam (1) & (2) eKTaj)an


1
1
1
1 81R 2 27 R 2

2
2
2
2
sin sin sin 3
p
p2

Bit

R2

dUcenH cot cot cot 3 9 . 2 1 .

p
1
1
1
b / vismPaBenHkayCasmPaBluHRtaEt

sin sin sin


2

eKTaj)an naM[ ABC CaRtIekaNsmgS .

eroberogeday lwm pln

- TMBr 51 -

KNitviTaCMuvijBiPBelak
lMhat;TI16
3 1 1
1 0 0
eK[m:aRTIs A 1 3 1 nig I 0 1 0
1 1 3
0 0 1
bgajfamansIVtcMnYnBitBIr (un ) nig ( vn ) Edlepgpat;
n IN : An un . I vn .A

EdleKnwgbBaak;tYTUeTA un nig vn

CaGnuKmn_n n .
dMeNaHRsay

bgajfa

n IN : An un . I v n .A

3 1 1
1 0 0
3 1 1
eKman A 1 3 1 0.0 1 0 1.1 3 1
1 1 3
0 0 1
1 1 3
eK)an A1 u1 .I v1 .A BitcMeBaH n 1 Edl u1 0 , v1 1 .
3 1 1 3 1 1 11 7 7
A 2 A.A 1 3 1 .1 3 1 7 11 7

1 1 3 1 1 3 7 7 11
0
0 21 7 7
10
A2 0
10
0 7 21 7 10.I 7.A

0
10 7 7 21
0
eK)an A 2 u2 .I v 2 .A BitcMeBaH n 2 .

eroberogeday lwm pln

- TMBr 52 -

KNitviTaCMuvijBiPBelak
]bmafavaBitcMeBaHRKb; n IN KW An un .I vn .A
eyIgnwgRsayfa An 1 un 1 .I vn 1 .A Bit .
eKman An 1 A.An A.(un .I vn .A) un .A.I vn .A 2
eday A.I A nig A 2 u2 .I v 2 .A
eK)an An 1 un .A vn (u 2 .I v 2 .A)
An 1 u 2 vn .I (un v 2 v n ).A
A n 1 un 1 .I v n 1 .A

Bit

Edl un 1 u 2 vn 10.vn nig vn 1 un v 2 vn un 7 vn


dUcenH n IN : An un . I vn .A
Edl (u n ) nig ( vn ) CasIVtcMnYnBitBIrkMNt;eday u1 0 , v1 1
nig u n 1 10.vn , vn 1 u n 7vn RKb; n IN .

bBaak;tYTUeTA

un

nig vn CaGnuKmn_n n

eKman un 1 10.vn , vn 1 un 7 vn RKb; n IN


tam vn 1 un 7 vn eK)an vn 2 un 1 7 vn 1
Et un 1 10vn enaH vn 2 7 vn 1 10vn
smIkarsmal;n vn 2 7 vn 1 10vn KW r 2 7r 10
b r 2 7r 10 0 manbs r1 2 , r2 5 .
tagsIVtCMnYy abn vvn 1 25vvn

eroberogeday lwm pln

n 1

- TMBr 53 -

KNitviTaCMuvijBiPBelak
an 1 vn 2 2 vn 1 7 vn 1 10 vn 2 v n 1 5an
eK)an b v 5v 7 v 10v 5v 2b
n 1
n 2
n 1
n 1
n
n 1
n
naM[ (an ) nig (bn ) CasIVtFrNImaRtmanersug 5 nig 2 erogKa .

eK)an an a1 .5n 1 nig bn b1 .2n 1 .


eday a1 v 2 2v1 7 2 5 nig b1 v 2 5v1 7 2 2
eKTaj an 5.5n 1 5n nig bn 2.2n 1 2n .
eK)anRbBn

v n 1 2 v n 5n

vn 1 5 vn 2n

dksmIkarBIrenHeK)an 3vn 5

naM[

5n 2n
vn
3

10 n
ehIy un 1 10vn 3 (5 2n )
naM[ un 103 (5n 1 2n 1 ) .
10 n 1
5n 2n
n 1
dUcenH un 3 (5 2 ) nig vn 3

eroberogeday lwm pln

- TMBr 54 -

KNitviTaCMuvijBiPBelak
lMhat;TI17
eK[ f CaGnuKmn_Cab; nig manedrIevelI IR EdlcMeBaHRKb; x, y IR
eKmanTMnak;TMng f ( x 2 y ) f (x)f (y ) nig f (x) 0 .
cUrkMNt;GnuKmn_ y f (x) .
dMeNaHRsay
kMNt;GnuKmn_ y f (x)
eKmanTMnak;TMng f ( x 2 y ) f (x)f (y )
eday f (x) 0 enaH f (y ) 0 nig f ( x 2 y ) 0
tagGnuKmn_ g(x) ln f (x) cMeBaHRKb; x IR
eK)an g( x 2 y ) ln f ( x 2 y ) eday f ( x 2 y ) f (x)f (y )

ehtuenH g( x 2 y ) ln f (x)f (y ) 12 ln f (x) ln f (y )


b g( x 2 y ) 12 g(x) g(y )
eFIVedrIeveFobnwg x kgsmPaB g( x 2 y ) 12 g(x) g(y )
edaycat;Tuk y CaGefrminGaRsynwg x eK)an
1 xy
1
xy
g' (
) g' ( x ) b g' (
) g' ( x )
2
2
2
2
eroberogeday lwm pln

- TMBr 55 -

KNitviTaCMuvijBiPBelak
yk x 0 eK)an g'( y2 ) g'(0)
eFIVGaMgetRkal g'( y2 )dy g'(o)dy g'(0)y k
tag z y2 enaH dy 2dz
eK)an 2 g'(z )dz g'(0)y k
1
k
k
2g( z ) g' (0)y k b g( z ) g' (0)y g' (0)z
2
2
2
yk a g'(0) nig b k2 eK)an g(z ) az b
b g(x) ax b eday g(x) ln f (x)
eK)an ln f (x) ax b naM[ f (x) e
dUcenH f (x) e Edl a , b IR .
ax b

ax b

eroberogeday lwm pln

- TMBr 56 -

KNitviTaCMuvijBiPBelak
lMhat;TI18
eK[ctuekaNe)a:g ABCD mYymanRCug AB a , BC b ,
CD c nig DA d .
P CacMnucmYyenAkgctuekaNehIy K , L , M , N CacMeNalEkg
ncMnuc P elIRCug [AB] , [BC] , [CD] , [DA] erogKa .
PL PM PN

h .
eKdwgfa PK
AB BC CD DA
k> cUrRsayfa h 2 22S 2 2 Edl S CapRkLan
a b c d
ctuekaN ABCD .
x>cUrbgajfa h 12
dMeNaHRsay
k>Rsayfa h 2 22S 2 2
a b c d

B
M
N

eroberogeday lwm pln

C
L

- TMBr 57 -

KNitviTaCMuvijBiPBelak
eKman

S S PAB S PBC S PCD S PDA


1
1
1
1
a.PK b.PL c.PM d.PN
2
2
2
2
PL PM PN
eday PK

h
AB BC CD DA
eKTaj PK ah , PL bh , PM ch , PN dh
eK)an S 12 a2h 12 b2h 12 c2h 12 d2h
dUcenH h 2 22S 2 2 .
a b c d
x>bgajfa h 12
eKman S S ABC SCDA 12 (ab sin B cd sin D)
eday sin B 1 nig sin D 1 enaHeK)an

1
c2 d 2
a2 b2
S (ab cd ) eday ab
; cd
2
2
2
a2 b 2 c2 d 2
eK)an S
4
tamsRmayxagelI h 2 22S 2 2
a b c d
a2 b 2 c2 d 2
2(
) 1
dUcenH h 2 2 4 2 2 2 .
a b c d

eroberogeday lwm pln

- TMBr 58 -

KNitviTaCMuvijBiPBelak
lMhat;TI19

eK[ctuekaNe)a:g ABCD mYymanRCug AB a , BC b


CD c nig DA d . O CacMnucmYyenAkgctuekaNenHEdl
OAB OBC OCD ODA 0 90o )
k> cUrRsayfa tan a2 b24Sc2 d2
Edl S CapRkLanctuekaN ABCD .
x> kMnt; edaydwgfa tan mantmFMbMput .
dMeNaHRsay

k> Rsayfa tan a2 b24Sc2 d2


D
C
O

eroberogeday lwm pln

- TMBr 59 -

KNitviTaCMuvijBiPBelak

tag OA x , OB y , OC z , OD t .
tamRTwsIbTkUsIunUskg OAB , OBC , OCD , ODA
y 2 a 2 x 2 2ax cos

(1)

z 2 y 2 b 2 2by cos

( 2)

t 2 z 2 c 2 2zc cos

( 3)

x 2 t 2 d 2 2td cos

( 4)

bUksmIkar (1) , (2) , (3) & (4) eKTaj)an


a2 b 2 c2 d 2
( 5)
cos
2(ax by cz dt )

m:ageTot S SOAB SOBC SOCD SODA


b S 12 (ax by cz dt ) sin
eKTaj sin ax by2S cz dt (6)
EckTMnak;TMng (6) nig (5) Gg nig GgeK)an
4S
.
tan 2
2
2
2
a b c d
x> kMnt; edaydwgfa tan mantmFMbMput
eKman S S ABC SCDA 12 (ab sin B cd sin D)
eday sin B 1 nig sin D 1 enaHeK)an
eroberogeday lwm pln

- TMBr 60 -

KNitviTaCMuvijBiPBelak
a2 b2
c2 d 2
1
; cd
S (ab cd ) eday ab
2
2
2
a2 b 2 c2 d 2
eK)an S
4
eday tan 2 24S 2 2
a b c d
a2 b 2 c2 d 2

1
tan
a2 b 2 c2 d 2

eKTaj
dUcenHedIm,I[ tan mantmFMbMputluHRtaEt 45o .

eroberogeday lwm pln

- TMBr 61 -

KNitviTaCMuvijBiPBelak
lMhat;TI20
cMeBaHRKb; x ; y 0 cUrRsaybBaak;vismPaB
x 2 x 1 y 2 y 1 x 2 x 1 y 2 y 1 2( x y )

(Kazakhstan NMO 2010)

dMeNaHRsay
RsaybBaak;vismPaB
x 2 x 1 y 2 y 1 x 2 x 1 y 2 y 1 2( x y )

eRCIserIsctuekaNe)a:g ABCD mYymanGgt;RTUg AC nig BD RbsBV


eroberogeday lwm pln

- TMBr 62 -

KNitviTaCMuvijBiPBelak
KaRtg;cMnuc I Edl IA x

, IC y , IB ID 1

nig AIB CID 60o . tamRTwsIbTkUsIunUseK)an


AB x 2 x 1 , CD y 2 y 1
AD x 2 x 1 , BC y 2 y 1

tamRTwsIbTGWEl AC.BD AB.CD AD.BC


eday AC AI IB x y

, BC BI IC 2

dUcenH
x 2 x 1 y 2 y 1 x 2 x 1 y 2 y 1 2( x y )

eroberogeday lwm pln

- TMBr 63 -

KNitviTaCMuvijBiPBelak
lMhat;TI21
eK[ a , b , c CacMnYnBitviCman . cUrRsaybBaak;fa
( 2a b c)2
( 2b c a )2
( 2c a b )2
2
2
8
2
2
2
2
2a (b c)
2b (c a )
2c (a b )
(USAMO 2003)

dMeNaHRsay
2
2
2
(
2
a

c
)
(
2
b

a
)
(
2
c

b
)
RsaybBaak; 2
2
2
8
2
2
2
2a (b c)

2b (c a )

2c (a b )

rebobTI1
tag

( 2a b c)2
( 2b c a )2
( 2c a b )2
T 2
2
2
2
2
2a (b c)
2b (c a )
2c (a b )2

yk s a b c enaHkenSam T Gacsresr
(a s )2
(b s ) 2
(c s ) 2
T 2
2
2
2
2
2a (a s )
2b (b s )
2c (c s )2

eKman

(a s )2
a 2 2as s 2
2
2
2
2a (a s )
3a 2as s 2
1 3a 2 6as 3s 2
. 2
3 3a 2as s 2
1
8as 2s 2
(1 2
)
2
3
3a 2as s

eroberogeday lwm pln

- TMBr 64 -

KNitviTaCMuvijBiPBelak
(a s )2
1
8as 2s 2
1
8as 2s 2
2

2
2
2
3 9a 6as 3s
3 ( 3a s )2 2s 2
2a (a s )

eday (3a s)2 0 enaH (3a s)2 2s2 2s2


8as 2s 2
8as 2s 2
4a
4a
1
1
b

2
2
2
s
abc
( 3a s ) 2s

eKTaj

2s

(a s )2
4
4a
(1)

2
2
3 abc
2a (a s )

RsaybMPWdUcKaxagelIEdreK)an
(b s )2
4
4b

( 2)
2
2

3
a
b
c
2b (b s )
(c s )2
4
4c

( 3)
2
2
3 abc
2c (c s )

bUkvismPaB (1) , (2) nig (3) eK)an

4 4 4 4a 4b 4c
44 8

3 3 3
abc
( 2a b c)2
( 2b c a )2
( 2c a b )2
dUcenH 2
2
2
8
2
2
2
2a (b c)
2b (c a )
2c (a b )
T

rebobTI2
tag

( 2a b c)2
( 2b c a )2
( 2c a b )2
T 2
2
2
2
2
2a (b c)
2b (c a )
2c (a b )2

yk x a b , y b c , z c a
eroberogeday lwm pln

- TMBr 65 -

KNitviTaCMuvijBiPBelak
eK)an

x z 2a b c

x y 2b c a
z y 2c a b

kenSam T GacsresrCa

nig

2a x z y

2b x y z
2c z y x

2( x z )2
2( z y )2
2( y x )2
T

2
2
2
2
( x z y ) 2y
( z y x ) 2x
( y x z )2 2z 2

tamvismPaB Cauchy Schwarz 2(u2 v 2 ) (u v )2


eK)an 2(x z y )2 2y 2 (x z y y )2 (x z )2
2( x z y )2 4y 2 ( x z )2 2y 2
1
( x z y )2 2y 2 ( x z )2 y 2
2
2( x z )2
2( x z )2
4

eKTaj
( x z y )2 2y 2 1 ( x z )2 y 2
2y 2
1
2
( x z )2
2y 2
y2
2
2
2
eday (x z ) 2(x z ) naM;[
2
2
(x z)
x z2

2y 2
x2 y 2 z 2
1

2
(x z )
x2 z 2

eKTaj)an

2( x z )2
4( x 2 z 2 )
(1)
2
2
2
2
2
( x z y ) 2y
x y z

eroberogeday lwm pln

- TMBr 66 -

KNitviTaCMuvijBiPBelak
RsaybMPWdUcxagelIEdreK)an
2( z y )2
4( z 2 y 2 )
2
( 2)
2
2
2
2
( z y x ) 2x
x y z
2( y x )2
4( y 2 x 2 )
2
( 3)
2
2
2
2
( y x z ) 2z
x y z

bUkvismPaB (1) , (2) , (3) eK)an


4( x 2 z 2 ) 4( z 2 y 2 ) 4( y 2 x 2 )
T
8
2
2
2
x y z
( 2a b c)2
( 2b c a )2
( 2c a b )2
dUcenH 2
2
2
8
2
2
2
2a (b c)
2b (c a )
2c (a b )

rebobTI3
tag

( 2a b c)2
( 2b c a )2
( 2c a b )2
T 2
2
2
2
2
2a (b c)
2b (c a )
2c (a b )2

eKmansmPaB (2u v )2 2(u v )2 3(2u2 v 2 )


edayyk u a nig v b c enaHeK)an

( 2a b c)2 2(a b c)2 3 2a 2 (b c)2

b (2a b c)2 3 2a2 (b c)2 2(a b c)2


EckGgTaMgBIrnwg 2a2 (b c)2 eK)an
( 2a b c)2
2(a b c)2
eyIg)an 2
3 2
2
2
2a (b c)

eroberogeday lwm pln

2a (b c)

- TMBr 67 -

KNitviTaCMuvijBiPBelak
kenSam T GacbMElgCa
(a b c )2
(b a c ) 2
(c a b )2
2
2
T 92 2
2
2
2

2
a
(
b
c
)
2
b
(
c
a
)
2
c
(
a
b
)

edIm,IRsayfa T 8 eyIgRtUvRsay[eXIjfa
(a b c )2
(b a c ) 2
(c a b )2
1

S 2
2a (b c)2 2b 2 (c a )2 2c2 (a b )2 2

eKman 2a2 (b c)2 2a2 b2 c2 2bc


eday 2bc b2 c2 enaH 2a2 (b c)2 2(a2 b2 c2 )
RsaydUcKaEdr 2b2 (c a)2 2(a2 b2 c2 )
ehIy 2c2 (a b)2 2(a2 b2 c2 )
(a b c )2 (b a c )2 (c a b ) 2
eKTaj S
2
2
2
2(a b c )

edaykenSam (a b c)2 (b a c)2 (c a b)2


esInwg 3(a2 b2 c2 ) 2(ab bc ca)
enaH S 32 ab2 bc2 ca2

a b c
a2 b 2 b 2 c2 c2 a2
eKman 2 2 2 ab bc ca
b a2 b2 c2 ab bc ca naM[ S 32 1 12 Bit .

eroberogeday lwm pln

- TMBr 68 -

KNitviTaCMuvijBiPBelak
rebobTI4
( 2a b c)2
( 2b c a )2
( 2c a b )2
Rsayfa 2
2
2
8
2
2
2
2a (b c)
2b (c a )
2c (a b )
bc
ca
ab
x
,
y
,
z
tag a

b
c

Edl

1
1
1
a
b
c

1
x1 y 1 z1 abc abc abc

vismPaBsmmUl

( 2 x )2 ( 2 y )2 ( 2 z )2

8
2
2
2
2 x
2 y
2 z

( 2 u )2 8
1
1
eyIgnwgRsayfa

RKb;
u0
2
3 1 u 3
2u
( 2 u )2 8
1
1
(u 5)(u 2)2
eKman


0 Bit
2
2
3
1

u
3
2u
3( 2 u )(1 u )
( 2 u )2
1
1 8

(*)
2
1

u
3
3
2u

ehtuenH
tam (*) eK)an

( 2 x )2 ( 2 y )2 ( 2 z )2
1
1
1

1 8
2
2
2
1

x
1

y
1

z
2 x
2 y
2 z

dUcenH

( 2a b c)2
( 2b c a )2
( 2c a b )2
2
2
8
2
2
2
2
2a (b c)
2b (c a )
2c (a b )

eroberogeday lwm pln

- TMBr 69 -

KNitviTaCMuvijBiPBelak
lMhat;TI22
cUrbgajfa 1 1 1 1 1 1

1
1

a b c d
ac bd
cMeBaHRKb;cMnYnBitviCman a , b , c , d .

dMeNaHRsay
eKman
X

nig

1
1 1

a b
Y

1
1 1

c d
1

evotNam

1962

ab
cd
abc abd acd bcd

ab cd
(a b )(c d )

(a c)(b d )
abcd

1
1

ac bd
(a c)(b d ) abc abd acd bcd

YX
abcd
(a b )(c d )

eK)an
bnab;BItRmUvPaKrYm rYcsRmYleK)an

(ad bc )2
YX
0 naM[ Y X
(a b )(c d )(a b c d )
dUcenH 1 1 1 1 1 1 1 1 1 .

a b c d
ac bd

eroberogeday lwm pln

- TMBr 70 -

KNitviTaCMuvijBiPBelak
lMhat;TI23
cUrbgajfa

1
1
1
4

( x y )2 ( y z )2 ( z x )2 xy yz zx

cMeBaHRKb;cMnYnBitminGviCmanxusKa x , y , z .
dMeNaHRsay
bgajfa 1

( x y )2

evotNam

2008

1
1
4

( y z )2 ( z x )2 xy yz zx

eday x , y , z CacMnYnBitxusKa nig minGviCmanenaHeKGacsntyk


zyx0 .
tag A 1 2 1 2 1 2
(x y )

(y z )

(z x)

nig B xy yz zx .
yk y x p , z x p q Edl p 0 , q 0
eK)an A 12 12 1 2
p

(p q )

nig B x(x p) (x p)(x p q) x(x p q)


3x 2 2( 2p q )x p 2 pq

edaysarEt x 0 enaH B p2 pq p(p q)


eroberogeday lwm pln

- TMBr 71 -

KNitviTaCMuvijBiPBelak
eK)an

1
1
1
AB 2 2
p(p q )
2
q
(p q )
p
p q p( p q )
p

p
pq
q2
1

q p(p q )
q
1

pq
p
q2

q
q
p( p q )
2(
)
p pq
q2
q2
p(p q )
2

p(p q )
q2
q2
p( p q )
q2
eday p(p q) 2 2 p(p q) . p(p 2 q) 2
q
q
eKTaj)an A B 2 2 4 naM[ A B4
dUcenH 1 2 1 2 1 2 xy yz4 zx .
(x y )
(y z )
(z x)
3x 2 2( 2p q )x 0
vismPaBenHkayCasmPaBluHRtaEt
p(p q ) q 2

eKTaj)an x 0 nig p(p q) q2 ehIy y p , z p q


enaH z y q tam p(p q) q2 eKTaj)an yz (z y )2
naM[ z 2 3yz y 2 0 eday z y enaH z 3 5 1
y

eroberogeday lwm pln

- TMBr 72 -

KNitviTaCMuvijBiPBelak
lMhat;TI24
cUrkMNt;tYTUeTAnsIVtEdlkMNt;eday
x 3 , x 4 nig x x nx cMeBaHRKb; n .
dMeNaHRsay
KNnatYtUeTA x
eKman
0

2
n 1

n 1

x0 3 0 3
x1 4 1 3

x 2 x 0 2 x1 9 4 5 2 3

]bmafa x n 2 , x n 3 Bit
eyIgnwgRsayfa x n 4
eKman x x nx
n 1

n 1

n 1

2
n 1

(n 2)2 n(n 3)

dUcenH

n 2 4n 4 n 2 3n
n4
xn n 3 .

eroberogeday lwm pln

- TMBr 73 -

KNitviTaCMuvijBiPBelak
lMhat;TI25
7
.
cUrkMNt;RKb;cMnYnBit x ebIeKdwgfa 128 27
18
6
dMeNaHRsay
kMNt;cMnYnBit x
7
eKman 128 27
tag
a 2 nig b 3 Edl a 0,b 0

18
6
smIkarkayCa
x

a3 b3
7

a 2b ab 2 6
(a b)(a 2 ab b 2 ) 7

ab(a b)
6
a 2 ab b 2 7

ab
6
6a 2 6ab 6b 2 7ab
6a 2 13ab 6b 2 0
(2a 3b)(3a 2b) 0

b ba 23 32
eKTaj)an

dUcenH x 1 , x 1 .
a 2

b 3

eroberogeday lwm pln

eday

a 2

b 3

- TMBr 74 -

KNitviTaCMuvijBiPBelak
lMhat;TI26
cUrKNnaplbUk
Sn

3
4
n2

...
1! 2! 3! 2! 3! 4!
n! (n 1)! (n 2)!

dMeNaHRsay
KNnaplbUk S
2
eKman S k ! (k k 1)!
(k )!
n

k 1

k2
k ! (k 1)! (k 2)!
k2

k ![1 (k 1) (k 1)(k 2)]


k2

k !(1 k 1 k 2 3k 2)
k2
1

k !(k 2)2 k !(k 2)


k 1
(k 2) 1

(k 2)!
(k 2)!
1
1

(k 1)! (k 2)!
dUcenH Sn kn1 ak 12 (n 1 2) ! .

tag

ak

eroberogeday lwm pln

- TMBr 75 -

KNitviTaCMuvijBiPBelak
lMhat;TI27 ( China 1983 )
eK[ f CaGnuKmn_kMNt;elIcenaH [0 ,1] edaydwgfa
f (0) f (1) 1 nig | f (a) f (b) | | a b |
cMeBaHRKb; a b kgcenaH [0 ,1] .
cUrbgajfa | f (a) f (b) | 12 .
dMeNaHRsay
bgajfa | f (a) f (b) | 12
-krNITI1
cMeBaH | a b | 12 enaHeK)an | f (a) f (b) | | a b | 12 Bit
-krNITI2
cMeBaH | a b | 12 enaHtamlkNqHeKGacsntfa a b
eKman | f (a) f (b) || f (a) f (1) f (0) f (b) |
tamvismPaBRtIekaNeK)an
| f (a) f (b) | | f (a) f (1) | | f (0) f (b) |

| f (a) f (b) | | a 1 | | 0 b | 1 a b 1 (a b)

1
2

dUcenH | f (a) f (b) | 12 .


eroberogeday lwm pln

- TMBr 76 -

KNitviTaCMuvijBiPBelak
lMhat;TI28 ( AIME 1988 )
cUrkMNt;KUncMnYnKt; (a ,b) edaydwgfa ax bx 1
Eckdac;nwg x x 1 .
dMeNaHRsay
kMNt;KUncMnYnKt; (a ,b)
tag p nig q Cabsrbs;smIkar x x 1 0 enaHtamRTwsIbTEvt
eK)an p q 1 nig pq 1 .
edIm,I[ ax bx 1 Eckdac;nwg x x 1 luHRtaEt p nig q
Cabsrbs;smIkar ax bx 1 0 Edr .
eK)an ap bp 1 nig aq bq 1
eKTaj)an q (ap bp ) q
b ap b q eRBaH pq 1
ehIy p (aq bq ) p b aq b p
ehtuenH (ap b) (aq b) p q
eKTaj)an a p p qq (p q)(p q )(p q )(p q )
eday p q 1 enaH p q (p q) 2pq 1 2 3
17

16

17

16

17

17

16

16

16

17

17

16

16

16

16

16

17

16

16

16

16

16

16

16

p 4 q 4 (p 2 q 2 )2 2p 2q 2 9 2 7
p 8 q 8 (p 4 q 4 )2 2p 4q 4 49 2 47

eroberogeday lwm pln

- TMBr 77 -

KNitviTaCMuvijBiPBelak
dUcenH a 1.3.7.47 987 .
ma:geTottam ap bp 1 nig aq bq
eK)an ap bp aq bq
eKTaj)an b pp qq .a Et a p p qq
17

17

enaH

16

16

17

17

17

17

16

16

16

16

16

16

p17 q17
(p16 p15q p14q 2 ... q16 )
b
pq
[(p16 q16 ) pq(p14 q14 ) p 2q 2 (p12 q12 ) ....
... p 7q 7 (p 2 q 2 ) p 8q 8 ]
[(p16 q16 ) (p14 q14 ) ... (p 2 q 2 ) 1]

tag S p q RKb; n 1 enaH S


eKman S p q
2n

2n

2n

2n 4

3,S 4 7 ,S 8 47

2n 4

2n 4

(p 2 q 2 )(p 2n 2 q 2n 2 ) p 2q 2 (p 2n q 2n )
3S 2n 2 S n

yktm n 3 ,4,5,6,7 ,8 CMnYskg S 3S S


eK)an S 18 ,S 47 ,S 123 ,S 322 , S 843
nig S 2207 .
eK)an b (2207 843 322 123 47 18 7 3 1)
b b 1597 .
dUcenH (a,b) (987 , 1597) .
2n 4

10

12

2n 2

2n

14

16

eroberogeday lwm pln

- TMBr 78 -

KNitviTaCMuvijBiPBelak
lMhat;TI29
edaHRsaykgsMNMucMnYnBitnsmIkar
x 3x x 2 .
3

dMeNaHRsay
edaHRsaysmIkar x 3x x 2
smIkarmannyluHRtaEt x 2 0 b x 2 .
-cMeBaH x 2 eKman x 4x x(x 2) 0 (i )
ehIy x x 2 (x 1)(x 2) 0 b x x 2 (ii)
bUkvismPaB (i ) & (ii) eK)an x 3x x 2
dUcenHsmIkar x 3x x 2 KanbscMeBaH x 2 .
-cMeBaH 2 x 2 eyIgGactag x 2 cos a Edl 0 a
smIkarGacsresrCa 8cos a 6cosa 2cosa 2
3

2(4cos 3 3cosa) 4cos 2

a
2

a
cos 3a | cos |
2

eday 0 a enaH cos a2 0


smIkarsmmUl cos 3a cos a2
eroberogeday lwm pln

- TMBr 79 -

KNitviTaCMuvijBiPBelak
a
a
2k 1 , 3a 2k 2
2
2
b a 4k51 , a 4k72 (k 1 ,k 2 )
eday 0 a enaHeKTaj)an a { 0 , 45 , 47 }
dUcenH x 2cos 0 2 , x 2cos 45 , x 2cos 47

eKTajbs

3a

eroberogeday lwm pln

- TMBr 80 -

KNitviTaCMuvijBiPBelak
lMhat;TI30 ( Korean Mathematics Competition 2000 )
cUrkMNt;RKb;cMnYnBit x Edlepgpat;smIkar
2x 3x 4x 6x 9x 1

dMeNaHRsay
kMNt;RKb;cMnYnBit x

2x 3x 4x 6x 9x 1
tag a 2x 0 nig b 3x 0

smIkarkaCa

a b a 2 ab b 2 1
1 a 2 b 2 a b ab 0
2 2a 2 2b 2 2a 2b 2ab 0
(1 2a a 2 ) (1 2b b 2 ) (a 2 2ab b 2 ) 0
(1 a)2 (1 b)2 (a b)2 0

eKTaj)an a b 1
ehtuenH 2 3 1 naM[ x 0 .
dUcenHsmIkarmanbsEtmYyKt;KW x 0 .
x

eroberogeday lwm pln

- TMBr 81 -

KNitviTaCMuvijBiPBelak
lMhat;TI31 ( China 1992 )
cUrbgajfa 16 1 17
k
dMeNaHRsay
bgajfa 16 1 17
80

k 1

80

k 1

eKman 2(

k 1 k)

2
2
1

k 1 k 2 k
k

80
1
2 ( k 1 k ) 16
k 1
k 1
k
2
2
1
ehIy 2( k k 1)

k k 1 2 k
k
eK)an k801 1 1 2k80 2 ( k k 1) 2 80 1 17
k
dUcenH 16 k801 1 17 .
k

eK)an

80

eroberogeday lwm pln

- TMBr 82 -

KNitviTaCMuvijBiPBelak
lMhat;TI32
z1 z 2 z 2 2
eK[ z1 , z 2 , z 3 CacMnYnkMupicEdl z12 z 22 z 32 3
z z z 4
1 2 3
cUrKNnatm S z z 1z 1 z z 1z 1 z z 1z 1
1 2
3
2 3
1
3 1
2

dMeNaHRsay
KNnatm S
eKman z z z
1

1 z 1z 2 z 3 1 (z 1 z 2 z 3 )
z 1z 2 1 z 1 z 2 (z 1 1)(z 2 1)

RsaybMPWdUcKaenHEdreK)an
z 2 z 3 z1 1 (z 2 1)(z 3 1), z 3 z1 z 2 1 (z1 1)(z 3 1)
z1 z 2 z 3 3
1
S

cyc (z 1)(z 1)
(z 1 1)(z 2 1)(z 3 1)
1
2

23
z1z 2 z 3 (z 1z 2 z 2 z 3 z 3 z 1 ) z1 z 2 z 3 1

2
2
2

10 2(z1z 2 z 2 z 3 z 3 z1 ) 10 (4 3)
9

eRBaH 2(z z
1

z 2 z 3 z 3 z 1 ) (z 1 z 2 z 3 )2 (z 1 2 z 2 2 z 3 2 )

eroberogeday lwm pln

- TMBr 83 -

KNitviTaCMuvijBiPBelak
lMhat;TI33
eK[ x , y , z CabIcMnYnBitviCmanEdl x y z 1 .
cUrkMNt;tmtUcbMputn 1 x x 1 y y 1 z z .
dMeNaHRsay
kMNt;tmtUcbMputn S 1 x x 1 y y 1 z z
eday x , y , z 0 nig x y z 1 enaHeKTaj 0 x, y, z 1
cMeBaH 0 t 1 tag f (t) t(1 t )
eK)an [f (t)] t (1 t ) b 8 f (t) 8t (1 t )
tamvismPaB AM GM eK)an
4

8 8

8 8

8t 8 8 8t 8
8
8
8
8
8
8
8t (1 t ) 8t .(1 t )(1 t )...(1 t )

9
8

9
9
8
8
eKTaj 8 f (t) 9 b f (t) 4 8 9 Et f (t) t(1 t 8 )

3
4
1
1
39
enaHeKTaj)anfa t(1 t 8 ) f (t) 8 (*)
x3
y3
z3
eKman S 1 x8 1 y 8 1 z 8

eroberogeday lwm pln

- TMBr 84 -

KNitviTaCMuvijBiPBelak
x4
y4
z4
eKGacsresr S x(1 x8 ) y(1 y 8 ) z(1 z 8 )

tamvismPaB (*) eKTaj)an


4
39 4
39
4
4
S
(x y z )
eRBaH
x4 y 4 z 4 1
8
8
x3
y3
z3
dUcenHtmtUcbMputn S 1 x8 1 y 8 1 z 8
4

KW

39 9 4 3

8
8
4

Smin

eroberogeday lwm pln

- TMBr 85 -

KNitviTaCMuvijBiPBelak
lMhat;TI34 ( Iran 1996 )
eK[bIcMnYnBitminGviCman a , b , c nigminsUnRBmKaBIr .
cUrRsaybBaak;fa

1
1
1 9

(ab bc ca)
2
2
2
(a b) (b c) (c a) 4

dMeNaHRsay
RsaybBaak;fa

1
1
1 9
(ab bc ca)
(*)

2
2
2
(b c )
(c a ) 4
(a b )
tag x a b c , y ab bc ca , z abc

eKmansmPaB
(a b )(b c)(c a ) (a b c)(ab bc ca ) abc

xy z

nig (a b)2 (a c)2 (x2 y )2 4x(xy z )


cyc

( x 2 y )2 4x( xy z ) 9
vismPaB (*) xagelIsmmUl y

2
( xy z )

4
smmUl 4x4y 17x2y 2 4y 3 34xyz 9z 2 0

xy(x3 4xy 9z) y(x4 5x2y 4y2 6xz) z(xy 9z) 0 (**)

eroberogeday lwm pln

- TMBr 86 -

KNitviTaCMuvijBiPBelak
tamvismPaB Schur cMeBaHRKb;cMnYnBitminGviCman x , y , z eKman
x( x y )( x z ) 0 x3 4xy 9z 0

(1)

cyc

x 2 ( x y )( x z ) 0 x4 5x2y 4y 2 6xz 0 ( 2)
cyc

tamvismPaB AM GM eKman
(a b c)(ab bc ca ) 9abc xy 9z 0 ( 3)

tam (1), (2) & ( 3) eKTaj)an (**) Bit .


dUcenH (ab bc ca) 1 2 1
(a b )

eroberogeday lwm pln

(b c )

1 9

2
(c a ) 4

- TMBr 87 -

KNitviTaCMuvijBiPBelak
lMhat;TI35
eK[ a , b , c CabIcMnYnBitminGviCman .
cUrRsayfa a3 b3 c3 3abc 2 ( b 2 c a)3 .
dMeNaHRsay
tamvismPaB AM GM eKman a3 b3 c3 3abc
enaH a3 b3 c3 3abc 0 .
ebI b 2 c a 0 enaHvismPaBxagelIBitCanic .
eyIg]bmafa b 2 c a 0 .

tag T a3 b3 c3 3abc 2( b 2 c a)3


yk b a 2x nig c a 2y enaH b c 2a 2x 2y
Edl x 0 , y 0 . eK)an
T a3 (a 2x)3 (a 2y)3 3a(a 2x)(a 2y) 2(x y)3

bnab;BIbRgYmrYceK)an
T 12a( x 2 xy y 2 ) 6( x y )( x y )2 6( x y )( x y )2
bc
bc 2
T 6(
a )(
) 0 Bit
2
2
dUcenH a3 b3 c3 3abc 2 ( b 2 c a)3 .

eroberogeday lwm pln

- TMBr 88 -

KNitviTaCMuvijBiPBelak
lMhat;TI36
eK[RtIekaN ABC mYymanRCug BC a , CA b , AB c
ehIymMukg A , B , C CamMuRsYcbmMuEkg .
tag S CapRkLan ABC
cUrRsaybBaak;fa 14 14 14 9 2 .
a

dMeNaHRsay
RsaybBaak;fa 14
a

16 S

1
1
9

b 4 c4 16 S 2

eKman S2 p(p a)(p b)(p c) Edl p a b2 c


tag x b2 c2 a2 , y c2 a2 b2 , z a2 b2 c2
Edl x , y , z 0 nig minGacmanBIrsUnRBmKa .
eK)an x y 2c2 , y z 2a2 , z x 2b2
ehIy 16S2 (a b c)(b c a)(c a b)(a b c)
4b 2c2 (b 2 c2 a 2 )2
( x y )( z x ) x 2 xy yz zx
vismPaB 14 14 14 9 2 GacbEmgeTACa
a
b
c
16 S
4
4
4
9

( x y )2 ( y z )2 ( z x )2 xy yz zx

eroberogeday lwm pln

- TMBr 89 -

KNitviTaCMuvijBiPBelak
( xy yz zx ) [

1
1
1
9

]
4
( x y )2 ( y z )2 ( z x )2

tamlkNHGUm:UEsneKGaceRCIserIsyk xy yz zx 1 enaHivsmPaB
xagelIsmmUl 4 (x y )2 (x z )2 9(x y )2 (y z )2 (x z )2
cyc

eday (x y )(x z ) x2 xy xz yz x2 1
ehIy (x y )(y z )(x z ) (x2 1)(y z )
x2 (y z ) y z
x( xy xz) y z
x(1 yz ) y z
x y z xyz

eK)an 4 (x2 1)2 9(x y z xyz)2


cyc

4[( x 2 1)2 ( y 2 1)2 ( z 2 1)2 ] 9( x y z xyz )2

4[(x4 y 4 z 4 ) 2( x2 y 2 z 2 ) 3] 9(x y z xyz)2 (*)

tag S x y z nig P xyz


Edl S a2 b2 c2 0 nig P 0 eRBaH x , y , z 0 .
eK)an S2 (x y z )2 x2 y 2 z 2 2
ehIy (S2 2)2 x4 y 4 z 4 2(x2y 2 y 2z 2 y 2z 2 )
eday xy yz xz 1 enaH (xy yz xz)2 1
eroberogeday lwm pln

- TMBr 90 -

KNitviTaCMuvijBiPBelak
b x2y 2 y 2z 2 x2z 2 2xyz(x y z ) 1
b x2y 2 y 2z 2 x2z 2 1 2S P
enaH (S2 2)2 x4 y 4 z 4 2(1 2S P)
naM[ x4 y 4 z 4 S4 4S2 4SP 2
vismPaB (*) xagelIsmmUl
4(S 4 4S 2 4SP 2 2S 2 4 3) 9(S P )2
4(S 4 2S 2 4SP 1) 9(S P )2
4S 4 8S 2 16SP 4 9(S P )2
9S 2 (4S 2 1)(S 2 4) 16SP 9(S P )2 (**)

-cMeBaH S 2 eKman (4S2 1)(S2 4) 16SP 0


naM[vismPaB (**) Bit
eRBaH 9S2 (4S2 1)(S2 4) 16SP 9S2 9(S P )2 .
-cMeBaH 0 S 2 vismPaB (**) Gacsresr
9S 2 ( 4S 2 1)(S 2 4) 16SP 9S 2 18SP 9P 2
(4S 2 1)(S 2 4) 34SP 9P 2 0

tag T (4S2 1)(S2 4) 34SP 9P 2


eyIgnwgRsayfa T 0 .
eKman (x y z )(xy yz xz) 9xyz tam AM GM
eday xy yz zx 1 enaH S 9P
eroberogeday lwm pln

- TMBr 91 -

KNitviTaCMuvijBiPBelak
ehIy T (4S2 1)(S2 4) P(S 9P ) 33SP
eK)an T (4S2 1)(S2 4) 33SP
tamvismPaB Schur eKman x2 (x y )(x z ) 0
cyc

eday x2 (x y )(x z ) x4 x2yz x3 ( y z )


eK)an ( x4 ) xyz ( x) x3 (y z )
cyc

cyc

cyc

eday (x4 ) S4 4S2 4SP 2

, xyz x SP

cyx

cyc

nig x3 (y z ) x2 (xy xz) x2 (1 yz )


cyc

cyc

cyc
2

x 2 xyz x S 2 SP
cyc

cyc

eK)an S4 4S2 5SP 2 S2 SP 2


6SP S 4 5S 2 4

ehtuenH

6SP (4 S 2 )(S 2 1)
11
T (4S 2 1)(S 2 4) 6SP
2
11
T (4S 2 1)(S 2 4) (4 S 2 )(S 2 1)
2
3
T (4 S 2 )(S 2 3)
2

eroberogeday lwm pln

- TMBr 92 -

KNitviTaCMuvijBiPBelak
eday 0 S 2 enaH 4 S2 0
ehIy S2 (x y z )2 3( xy yz zx) 3 enaH S2 3 0
eKTaj)an T 32 (4 S2 )(S2 3) 0 Bit .
srubmkeK)an (xy yz zx) [
dUcenH

1
1
1
9

a4 b4 c4 16 S 2

eroberogeday lwm pln

1
9
1
1
]

4
( x y )2 ( y z )2 ( z x )2

- TMBr 93 -

KNitviTaCMuvijBiPBelak
lMhat;TI37 Turkey 2007
eK[bIcMnYnBitviCman a , b , c Edl a b c 1 . cUrRsaybBaak;fa
1
1
1
1

ab 2c 2 2c bc 2a 2 2a ca 2b 2 2b ab bc ca

dMeNaHRsay
CadMbUgeyIgnwgRsayfa

1
ab

ab 2c 2 2c (ab bc ca)2

smmUl (ab bc ca)2 ab(ab 2c2 2c)


smmUl b2c2 c2a2 2abc(a b c) 2abc2 2abc
eday a b c 1 enaH b2c2 c2a2 2abc 2abc2 2abc
smmUl b2c2 c2a2 2abc2 c2 (a b)2 0 Bit
1
ab

ehtuenH
(1)
2
2
ab 2c 2c (ab bc ca)
1
bc

( 2)
2
2
bc 2a 2a (ab bc ca )
1
ca

( 3)
2
2
ca 2b 2b (ab bc ca )

bUkvismPaB (1) , (2) & (3) eK)an


1
1
1
1

ab 2c 2 2c bc 2a 2 2a ca 2b 2 2b ab bc ca

eroberogeday lwm pln

- TMBr 94 -

KNitviTaCMuvijBiPBelak
lMhat;TI38
eK[ a , b , c CabIcMnYnBitviCman . cUrRsaybBaak;fa
a2
2a 2 ab b 2

dMeNaHRsay

b2
2b 2 bc c2

eyIgBinitGnuKmn_ f (x)

x2

c2
2c2 ca a 2

abc
2

RKb; x 0

2x x 1
vismPaBxagelIsmmUl bf ( ba ) cf ( bc ) af ( ac ) a b2 c
2

.
edIm,IRsaybBaak;vismPaBenHeyIgRtUvkMNt;GnuKmn_lIenEGmYyEdl
f ( x ) x (*) ehIy nig CaBIrcMnYnBitEdlbMeBjlkxN (*)
BitcMeBaHRKb; x 0 .
edaysaEtvismPaBBitcMeBaH a b c enaHeyIgnwgkMNt;rk nig
EdleFIV[ExSekag (c) : y f (x) b:Hnwg (d) : y x Rtg; x 1
eBalKWRtUv[ f (1) nig f ' (1) .
eday f (1) 12 enaH 12
eKman f ' (x)

2x 2x 2 2x 1

eroberogeday lwm pln

4x 1

2 2x 2 2x 1
2x 2 2x 1

.x 2

- TMBr 95 -

KNitviTaCMuvijBiPBelak
5
11
11
eK)an f ' (1) 4 4 16
enaH 16
ehIy 12 163
11x 3
x2
11x 3

ehtuenH f (x) 16 b 2
(**)
16
2x x 1
-ebI 0 x 113 enaHvismPaB (**) BitCaniceRBaHGgTI1CakenSamviCman
CanicRKb; x 0 ehIyGgTIBIrGviCman .
-ebI x 113 enaHvismPaB (**) Gacsresr
2
2
2
2
2

x
(
11
x
3
)
(
x

1
)
(
14
x
39x 9)

0
2

2
6
25
256( 2x x 1)
2x 2x 1
vaBitcMeBaHRKb; x 113 eRBaH 14x2 39x 9 0 .
tam (**) eKCMnYs x eday ba , bc , ac eK)an
a
b
c 11a 3b 11b 3c 11c 3a
bf ( ) cf ( ) af ( )
b
c
a
16
8a 8b 8c a b c

Bi
t
16
2
4

dUcenH

a2
2a 2 ab b 2

b2
2b 2 bc c2

eroberogeday lwm pln

c2
2c2 ca a 2

abc
2

- TMBr 96 -

KNitviTaCMuvijBiPBelak
lMhat;TI39
eK[ a , b , c CabIcMnYnBitxusKa .
a2
b2
cUrRsayfa

2
2
(b c )

(c a )

c2
2
2
(a b )

dMeNaHRsay
eKman x2 y 2 z 2 (x y z )2 2(xy yz zx)
eday (x y z )2 0 RKb;cMnYnBit x , y , z enaHeK)an
x 2 y 2 z 2 2( xy yz zx ) (*)
yk x b a c , y c b a , z a c b enaHeK)an
ab
bc
ac
xy yz zx

(b c)(c a ) (a b )(c a ) (a b )(b c)


ab(a b ) bc(b c) ac(c a )

(a b )(b c)(c a )

b(c a )(b c a ) ac(c a )


(a b )(b c)(c a )
(c a )(b c)(b a )

1
(a b )(b c)(c a )

tamTMnak;TMng (*) eKTaj)an x2 y 2 z 2 2(1) 2


a2
b2
c2
dUcenH

2 .
2
2
2
(b c )

(c a )

eroberogeday lwm pln

(a b )

- TMBr 97 -

KNitviTaCMuvijBiPBelak
lMhat;TI40
cUrRsaybBaak;vismPaB
a 2 (1 b )2 b 2 (1 c)2 c2 (1 a )2

cMeBaHRKb;cMnYnBit a , b , c .
dMeNaHRsay
tamvismPaB Minkowsky eKman

3 2
2

x12 y12 x 22 y 2 2 x 3 2 y 32 ( x1 x 2 x 3 )2 ( y1 y 2 y 3 )2

yk x1 a , x2 b , x3 c
nig y1 1 b , y 2 1 c , y 3 1 a nig s a b c
eK)an (x1 x2 x3 )2 (y1 y 2 y 3 )2 s2 (3 s)2 92
eRBaH s2 (3 s)2 2s2 6s 9 12 [(2s 3)2 9] 92
dUcenH
a 2 (1 b )2 b 2 (1 c)2 c2 (1 a )2

eroberogeday lwm pln

3 2
2

- TMBr 98 -

KNitviTaCMuvijBiPBelak
lMhat;TI41
eK[RtIekaN ABC mYyEkgRtg; C . D nig E CacMNucBIreRCIserIs
enAelIGIub:UetnUs Edl BC BD nig AC AE .
F nig G CacMeNalEkgn D nig E elIRCug AC nig BC erogKa .
cUrRsaybBaak;fa DE DF EG .
dMeNaHRsay
RsaybBaak;fa DE DF EG

eyIgsg;km<s; CN rYcPab; CD nig CE .


eKman BC BD enaH BCD CaRtIekaNsm)atkMBUl B
eK)an BCD BDC Et BCD BCN NCD
nig BDC DCA A
eroberogeday lwm pln

- TMBr 99 -

KNitviTaCMuvijBiPBelak
ehtuenH BCN NCD DCA A (1)
eday CN AB enaH BCN A ( 2) mMubMeBjnmMu NCA
tam (1) nig (2) eKTaj)an NCD DCA
ehIy CND CFD 90o enaHeKTajRtIekaN CND nig CFD
CaRtIekaNEkgb:unKa . eKTaj)an DN DF .
RsaybMPWtamrebobdUcKaeK)an EN EG .
dUcenH DE DN NE DF EG .

eroberogeday lwm pln

- TMBr 100 -

KNitviTaCMuvijBiPBelak
lMhat;TI42
eKyk D CacMnucmYynRCug BC rbs;RtIekaN ABC ehIy E nig F
CacMeNalEkgn B nig C elI AD .
ebI R CacMnuckNaln BC enaHRsayfa RE RF
dMeNaHRsay
Rsayfa RE RF

bnay CF FG nig BE EH rYcPab; AG , BG , AH nig CH .


eKman BE AE nig BE EH sMNg;enaH ABE AHE
eK)an AB AH nig BAE HAE .
eroberogeday lwm pln

- TMBr 101 -

KNitviTaCMuvijBiPBelak
RsaydUcKaEdreK)an ACF AGF
eK)an AC AG nig CAF GAF .
eFIVpldk CAF BAF GAF AHE
eKTaj)an BAG CAH enaH AGB ACH
Cavi)akeKTaj BG CH .
kgRtIekaN CBG eKman R nig F CacMnuckNaln CB nig CG
eK)an RF 12 BG )atmFm
dUcKaEdrkgRtIekaN CBH eK)an RE 12 CH .
eday BG CH dUcenH RE RF .

eroberogeday lwm pln

- TMBr 102 -

KNitviTaCMuvijBiPBelak
lMhat;TI43
eK[kaer ABCD mYy . E CaRbsBVrvagGgt;RTUgTaMgBIr ehIy N CacMnuc
mYysitelI AE .
cUrRsayfa AB2 BN 2 AN.NC nig AN2 NC2 2BN 2
dMeNaHRsay
Rsayfa AB2 BN 2 AN.NC

eKman AC BD Ggt;RTUgnkaer enaH AE BE


tamRTwsIbTBItaKrcMeBaH AEB nig NEB
AB 2 AE2 EB 2 ( AN NE)2 EB 2 (1)
BN 2 NE2 EB 2 ( 2)

eroberogeday lwm pln

- TMBr 103 -

KNitviTaCMuvijBiPBelak
dksmIkarBIrenHGgnigGgeK)an
AB 2 BN 2 AN 2 2AN.NE AN.( AN 2NE)

Et AN 2NE AE NE NE EC NC
dUcenH AB2 BN 2 AN.NC .
Rsayfa AN2 NC2 2BN 2
eKman AN2 ( AE NE)2 AE2 NE2 2AE.NE (3)
ehIy NC2 (NE EC)2 (NE AE)2 eRBaH EC AE
NC2 NE2 AE2 2 AE.NE (4)

bUksmIkar (3) nig (4) eK)an


AN 2 NC2 2NE2 2 AE2

Et AE BE

dUcenH AN2 NC2 2(NE2 BE2 ) 2BN 2 .

eroberogeday lwm pln

- TMBr 104 -

KNitviTaCMuvijBiPBelak
lMhat;TI44
eK[ ABC CaRtIekaNmYyehIy D CaeCIgnkm<s;KUsBIkMBUl A .
yk E nig F sitenAelIbnat;kat;tam D edaydwgfa AE Ekgnwg BE
ehIy AF Ekgnwg CF Edl E nig F xusBI D .
yk M nig N CacMNuckNalnGgt; BC nig EF erogKa .
cUrRsayfa AN Ekgnwg NM ?
dMeNaHRsay
Rsayfa AN Ekgnwg NM
A
F

Q
N

E
B

eroberogeday lwm pln

- TMBr 105 -

KNitviTaCMuvijBiPBelak
tag (w1 ) nig (w 2 ) CargVg;manGgt;piterogKa AB nig AC .
eK)an E (w1 ) eRBaH AE BE ehIy F (w 2 ) eRBaH AF CF
tag P nig Q CacMnucRbsBVrvagbnat; ( AN) CamYy (w1 ) nig (w 2 )
erogKa .
cMnuc N enAeRkArgVg; (w1 ) enaHeK)an NE.ND NQ.NA (1)
eRBaH N CacMNuckNaln EF .
cMeBaHrgVg; (w 2 ) eKman NF.ND NP.NA (2)
tam (1) & (2) eKTaj)an NQ.NA NP.NA b NQ NP
eday MB MC enaHeK)an MN // PC ehIyeday AP PC
dUcenH AN NM .

eroberogeday lwm pln

- TMBr 106 -

KNitviTaCMuvijBiPBelak
lMhat;TI45
rkGnuKmn_ f : IR IR Edlepgpat;
x, y IR : f ((x y)2 ) x 2 2yf (x) f (y)

dMeNaHRsay
rkGnuKmn_ f
yk y x eK)an f (0) x 2 2xf ( x ) f ( x )2 ( x f ( x ))2
ebI x 0 enaH f (0) f 2 (0) naM[ f (0) 0 b f (0) 1
-krNI f (0) 0 eK)an ( x f ( x ))2 0 enaH f ( x ) x
epgpat;
f (( x y )2 ) ( x y )2
x 2 2 yf ( x ) f ( y )2 x 2 2 yx y 2 ( x y )2

eK)an f (( x y)2 ) x 2 2yf ( x ) f ( y) 2 ( x y)2 Bit


dUcenH f ( x ) x RKb; x IR
-krNI f (0) 1 eK)an ( x f ( x ))2 1 enaH f ( x ) x 1
b f ( x ) x 1 .
epgpat;
-cMeBaH f (x) x 1

eroberogeday lwm pln

- TMBr 107 -

KNitviTaCMuvijBiPBelak
f ((x y)2 ) (x y)2 1
x 2 2yf (x) f (y) x 2 2y(x 1) (y 1)2
2

(x y)2 1

eK)an f (( x y)2 ) x 2 2yf ( x ) f ( y) 2 ( x y)2 1 Bit


dUcenH f ( x ) x 1 RKb; x IR
-cMeBaH f ( x ) x 1
f ((x y)2 ) (x y)2 1
x 2 2yf (x) f (y) x 2 2y(x 1) (y 1)2
2

(x y)2 1

eK)an f (( x y)2 ) x 2 2yf ( x ) f ( y) 2


dUcenH f ( x ) x 1 minEmnCacemIy .
srubmkeK)an f ( x ) x b f ( x ) x 1 RKb; x IR .

eroberogeday lwm pln

- TMBr 108 -

KNitviTaCMuvijBiPBelak
lMhat;TI46
cMeBaHRKb;cMnYnBitviCman a ,b ,c cUrRsaybBaak;fa
3

(a 2 b2 )(b2 c2 )(c2 a 2 ) 3
(a b)(b c)(c a )
abc
8
6

dMeNaHRsay
bgajfa

(a 2 b2 )(b2 c2 )(c2 a 2 ) 3
(a b)(b c)(c a )
abc
8
x 2 y2
CadMbUgeyIgRtUvRsayfaRKb; x , y 0 eK)an x y 2 xy
eKman x y 2 xy enaH ( x y)2 4xy b ( x y)2 4xy 0
3

elIkCakaer [( x y)2 4xy]2 0


( x y )4 8xy( x y )2 16 x 2 y 2 0
( x y )4 8xy[( x y )2 2 xy] 8xy( x 2 y 2 )
( x y )2
x 2 y2
x 2 y2
eKTaj ( x y) 4 xy 2 b 2 2 xy 2
2
x 2 y2
x 2 y2
2 ( x y)
eday 2 xy. 2 ( 2 xy ) 2
2
( x y )2
x 2 y2
2 ( x y)
eK)an 2 ( 2 xy) 2
2

eroberogeday lwm pln

- TMBr 109 -

KNitviTaCMuvijBiPBelak
x 2 y2
naM[ x y 2 xy .
a b p ab
tamvismPaBxagelI eKTaj b c q bc

a c r ac
a 2 b2
b2 c2
a 2 c2
Edl p 2 , q 2 nig r 2
eK)an (a b)(b c)(c a ) ( p ab )( q bc )( r ac )

b (a b)(b c)(c a ) pqr m n abc


Edl m acpq bcpr abqr 33 a 2b2c2p2q 2r 2
nig n abc2p a 2bcq ab2cr 33 a 4b4c4pqr
eday pqr m n abc (6 pqr 3 abc )3
eKTaj (a b)(b c)(c a ) (6 pqr 3 abc )3
b 3 (a b)(b c)(c a ) 6 pqr 3 abc
(a 2 b2 )(b2 c2 )(c2 a 2 ) 3
3
6
dUcenH (a b)(b c)(c a)
abc
8

eroberogeday lwm pln

- TMBr 110 -

KNitviTaCMuvijBiPBelak
lMhat;TI47 Greece National Olympiad 2011
eK[ a , b , c CacMnYnBitviCmanEdlmanplbUkesI 6 .
cUrkMNt;tmGtibrman S a 2bc b 2ca c 2ab
dMeNaHRsay
kMNt;tmGtibrman S 3 a 2 2bc 3 b2 2ca 3 c2 2ab
tag u 3 a 2 2bc , v 3 b2 2ca nig w 3 c2 2ab
eKman u3 v3 w 3 a 2 2bc b2 2ca c2 2ab (a b c)2
Ettamsmtikm a b c 6 enaH u3 v3 w 3 36 (1)
ehIy S u v w (2)
cMeBaHRKb; x 0 eyIgeRCIserIs f ( x ) x3
eKman f ' ( x ) 3x 2 nig f '' ( x ) 6x 0 enaHtamvismPaB Jensen eK)an
3

f ( u ) f ( v ) f ( w ) 3f (

eK)an

uvw
) , u , v, w 0
3
3

1
uvw
3
u v w 3
( u v w ) (3)
3
9

S3
36
9

naM[ S 3 3 12
tamTMnak;TMng (1), (2) & (3) eK)an
dUcenHtmGtibrman S 3 a 2 2bc 3 b2 2ca 3 c2 2ab
esInwg Smax 3 3 12 EdlRtUvnwg a b c 2 .

eroberogeday lwm pln

- TMBr 111 -

KNitviTaCMuvijBiPBelak
lMhat;TI48 USAMO 1989
cMeBaHRKb;cMnYnKt;viCman n eK[
1 1
1
....
n
2 3
Tn S1 S 2 S 3 .... Sn

Sn 1

Un

T1 T2 T3
T

.... n
n1
2
3
4

cUrkMNt;edayeFIVdMeNaHRsay nUvcMnYnKt; 0 a, b, c, d 1000 000


edaydwgfa T1988 aS1989 b nig U1988 cS1989 d .
dMeNaHRsay
kMNt;cMnYnKt; a , b , c , d
eyIgnwgRsaytamkMeNInfaRKb; n 2 : Tn 1 nSn n
-cMeBaH n 2 : T1 2S2 2 2(1 12 ) 2 1 S1 Bit
-]bmafa Tn 1 nSn n Bit
eyIgnwgRsayfa Tn (n 1)Sn 1 (n 1) Bit
eKman Tn S1 S2 ... Sn 1 Sn Tn 1 Sn
Tn nS n n Sn (n 1)Sn n
eday Sn 1 Sn n 1 1 enaH Sn Sn 1 n 1 1

eroberogeday lwm pln

- TMBr 112 -

KNitviTaCMuvijBiPBelak
eK)an Tn (n 1)(Sn 1 n 1 1) n (n 1)Sn 1 (n 1) Bit
dUcenH n 2 : Tn 1 nSn n
yk n 1989 eK)an T1988 1989 S1989 1989
eday T1988 aS1989 b enaHeKTaj a 1989 , b 1989 .
n
T1 T2 T3
Tn
ehIy Un 2 3 4 ... n 1 ( kTk 1)
k 1

Un

(k 1)Sk 1 (k 1)

k 1
k 1
n

(Sk 1 1)

k 1

S 2 S 3 .... Sn 1 n
Tn 1 S1 n
(n 1)Sn 1 (n 1) 1 n
(n 1)Sn 1 2(n 1)

yk n 1988 eK)an U1988 1989S1989 3978


eday U1988 cS1989 d enaH c 1989 , d 3978
dUcenH a 1989 , b 1989 , c 1989 , d 3978 .

eroberogeday lwm pln

- TMBr 113 -

KNitviTaCMuvijBiPBelak
lMhat;TI49 (Japan Mathematical Olympiad Finals 2010)
eK[ x , y , z CacMnYnBitviCman . cUrbgajfa
1 yz zx 1 zx xy 1 xy yz

1 .
(1 x y) (1 y z) (1 z x)
2

dMeNaHRsay
tamvismPaB Cauchy Schwarz
(a1b1 a 2b 2 a 3b 3 )2 (a12 a 2 2 a 3 2 )(b12 b 2 2 b 3 2 )

x
y
,a 3 yz ,b 3
z
z
eK)an (1 x y)2 (1 xz yz)(1 xz yz )
eKTaj)an (11 xzx y)yz2 x yz z (1)
RsaydUcKaEdr (11 zxy z)xy2 x xy z (2)
1 xy yz
y

(3)
nig (1
z x)2 x y z

yk a

b1 1 ,a 2 xz ,b 2
1

eFIVplbUkvismPaB (1), (2) & (3) eK)an

1 yz zx 1 zx xy 1 xy yz x y z

1
(1 x y)2 (1 y z)2 (1 z x)2 x y z
1 xy yz
1 .
dUcenH (11 yzx yzx)2 (11 zxy zxy)2 (1
2
z x)

eroberogeday lwm pln

- TMBr 114 -

KNitviTaCMuvijBiPBelak
lMhat;TI50 China Team Selection Test 2002
eK[sIVt a 1, a 5 nig a a a
1

n 1

n 1

an a
2

2
n 1

n 2

cUrkMNt;tYTUeTAnsIVt {a } CaGnuKmn_n n .
dMeNaHRsay
kMNt;tYTUeTAnsIVt {a } CaGnuKmn_n n
eKman a a a
n 2
n

n 1

eK)an
1
an2 1
1
an2 1
1

n 1

an 2 an2 1 1

an 2 an2 1 1

an 2an2 1

1
1
1

an 2 an2 1 an 2an2 1

1
an2 1

1
1
1
)
(1
)

an 2 an2 1
an 2

1
1
)(1
)

an2 1
an2
an2 1
b 1 a12 (1 a12 )(1 a12 ) (*)
n 2
n
n 1
tag bn ln(1 a12 ) enaH bn1 ln(1 a12 )
n
n 1
1

(1
(1

eroberogeday lwm pln

- TMBr 115 -

KNitviTaCMuvijBiPBelak
ehIy

bn 2 ln(1

1
an2 2

) (**)

yk (*) CMnYskg (**) eKTTYl)an b b b


smIkarsmal; q q 1 0 manbs q 1 2 5
n 2

n 1

, q2

1 5
2

1 5 n
1 5 n
)

(
) Edl , kMNt;dUcxageRkam
n
2
2
cMeBaH n 1 b1 . 1 2 5 . 1 2 5
cMeBaH n 2 : b2 3 2 5 3 2 5
26
eday b1 ln(1 a12 ) ln 2 nig b2 ln(1 a12 ) ln 25
1
2
1 5
1 5

ln 2

2
eK)an 2
3 5 3 5 ln 26
2
2
25

eKTaj 12 ln 2 513 1 ln 100

3
5

1 2 13 1 100
nig 2 ln 5 ln 3
5

ehIy bn ln(1 a12 ) eKTaj an b1n .


n
e 1

eK)an b

eroberogeday lwm pln

- TMBr 116 -

KNitviTaCMuvijBiPBelak
lMhat;TI51 China Team Selection Test 2006
eK[ x , x ,...., x CacMnYnBitviCmanEdl x 1 . cUrRsayfa
n

i 1

xi

n
1
i1
1 xi

n2

n1

dMeNaHRsay
tag 1 x y eKTaj x
i

vismPaB
n

xi

i 1

i 1

n
y i 1
i 1

i 1

y i 1 nig

n1

yi

i 1

n
1
n2
i1

1 xi
n1

1
n2

yi
n1

tamvismPaB Cauchy Schwarz a b


eKyk a 1 ,a y 1 i 2
1

ehIy

i1

smmUl

i 1

Edl y

(a i

i 1

). (b i 2 )
i 1

b1 y 1 1 , b i

1
, i 2
n

eK)an

n 1
1
1 n

. y i 1 (y i 1) y 1 1
i2 n
n

n i2

eday (y 1) n 1 y (n 1) 2 y
ehIy n1 n n 1 enaHeK)an
n

i2

i2

eroberogeday lwm pln

- TMBr 117 -

KNitviTaCMuvijBiPBelak
n 1
1 2n
1
1 n

(y
1)
y
1
(
y
)(y
)

i
i
i

i
2
i
2
n
n
n
n

y 12

2(n 1)
2n 1
y1
n
n2

eKTaj in1 1 . y i 1 y12 2(nn 1) y1 2nn2 1


n

b in1 y i 1 n y12 2(nn 1) y1 2nn2 1


b 1 in1 y i 1 n y1 2nn 2 2nn2 1 . y1
y1
1
2

dUcKaEdreKTaj)an
1
y2

i 1

yi 1 n y2

2n 2 2n 1 1
.

2
y2
n
n

1
yn

i 1

yi 1 n yn

2n 2 2n 1 1
.

n
n2 yn

edayeFIvviFIbUkGg nig GgnbNavismPaBxagelIeK)an


n

i 1

yi 1 .

i 1

n
1
2n 2 2n 1 1
n yi

. (*)
2
i
1

n
n
yi
yi

tamvismPaB Cauchy Schwarz eKman


n

i 1

yi

2n 2 2n 1 1
. n

2
n2 y i

eroberogeday lwm pln

(y i

i 1

2n 2 2n 1 1

. )
n
n2 y i

- TMBr 118 -

KNitviTaCMuvijBiPBelak
eday ( y
n

i 1

eK)an
n

i 1

ehIy

2n 2
) (n 1) 2n 2 n 1
n

yi

2n 2 2n 1 1
2n 1 n 1

n
n

.
2
n2 y i
n 2 i 1 y i

1 (1 1 ... 1)2
n2

n
i 1 y
n1
yi
i
n

i 1

eKTaj
n

i 1

2n 2 2n 1 1
2n 1 n 2
yi

. n n 1
.
2
n2 y i
n2 n 1

yi

i 1

2n 2 2n 1 1
n

n
.
(**)
2
n2 y i
n1

n 1
n2
tam (*) nig (**) eKTaj i1 y i 1 i1
yi
n1

n
n
1
n2
dUcenH i1 xi i1
.

1 xi
n1

Bit

eroberogeday lwm pln

- TMBr 119 -

KNitviTaCMuvijBiPBelak
lMhat;TI52 China Team Selection Test 2005
eK[ a ,b ,c CabIcMnYnBitminGviCman Edl ab bc ca 13 .
1
1
1

3 .
cUrbgajfa a bc
1 b ca 1 c ab 1
dMeNaHRsay
a
b
c
tag S a bc

1 b ca 1 c ab 1
2

1
1
1

a 2 bc 1 b 2 ca 1 c 2 ab 1
a2
b2
c2
eKman S a3 abc a b 3 cab b c3 abc c
tamvismPaB Cauchy Schwarz eK)an

nig

(a b c)2
S 3
a b 3 c 3 3abc a b c

Et a b c 3abc (a b c)(a b c ab bc ca)


bc
1
eK)an S a b ca ab
eday
ab bc ca
bc ca 1
3
3

abc
1
.

2
b 2 c 2 2ab 2bc 2cc a b c
ca ab bc ca ab bc ca
ma:geTot T3 aba2 bc
2
2
bc 1
b ca 1
c ab 1
ca a(a b c)
1
ehIy aba2 bc
2
2
1
bc 1
a bc 1 a bc 1

enaH S a

eroberogeday lwm pln

- TMBr 120 -

KNitviTaCMuvijBiPBelak
eK)an

T
ab bc ca
2
3 cyc a bc 1
a(a b c)
1
[ 2
2
1]
cyc a bc 1
a bc 1
(a b c)S T 3 (a b c).

1
T3
abc

eRBaH S a b1 c sRmayxagelI
eKTaj T T3 2 b T 3 .
dUcenH a

1
1
1

3
2
2
2
bc 1 b ca 1 c ab 1

eroberogeday lwm pln

- TMBr 121 -

KNitviTaCMuvijBiPBelak

eroberogeday lwm pln

- TMBr 122 -

KNitviTaCMuvijBiPBelak
CMBUkTI3

lMhat;Gnuvtn_
!> eK[sIVtncMnYnBit (x ) kMNt;eday
n

cMeBaHRKb; n 1 cUrRsaybBaak;fa
1

1
2n 1

x1 2

xn 1
x
x
n 1
n

1
1 1
1
.... 1 n
x1 x 2
xn
22

2
@> eK[ a , b ,c CacMnYnBitviCmanEdl abc 1
1
1
1

1 .
ab1 bc1 ca1

.
. cUrbgajfa

#> cUrkMNt;tmGb,brmanplbUk

1
1
1
Sn log x1 (x 2 ) log x2 (x 3 ) ... log xn (x1 )
4
4
4
Edl x1 , x2 , ...., xn CacMnYnBitncenaH ( 14 ,1) .
$> cUrkMNt;GnuKmn_ f : IR IR ebIeKdwgfa
f (xf (x) f (y)) (f (x))2 y

eroberogeday lwm pln

cMeBaHRKb; x, y IR .

- TMBr 123 -

KNitviTaCMuvijBiPBelak
3x y

3
2
2

%> edaHRsayRbBnsmIkar xx 3yy


y
0

x2 y 2
n
1
^> KNnaplbUk k0 (n k)!(n
k)!

&> edaHRsaysmIkar 2(2 1)x (2 2)x 2


*> cUrbgajfa 12 . 43 .... 2n2n 1 1
3n
cMeBaHRKb;cMnYnKt;viCman n .
x

x2

x 0 a , x1 b
(> eK[sIVt x 1 (x 1 )
n 1 2 n 1 x

n
cUrbgajfa a.b 1 .

n 1

CasIVtxYb Periodic .

!0> cUrkMNt;RKb;cemIyBitrbs;smIkar 2 3
!!> eK[ {a } CasIVtmYyEdl a 2 , a
n

x 1

6x x 2 .
an 1

( n IN )
n 1
2 an
x

cUrkMNt;rUbmnGiucBIsIutmYyn (a ) .
!@> eK[ a ,a , ....,a CasIVtncMnYnBitEdl a
Edl n 1 . cUrbgajfa a (2 , 1) .
n

n 1

an 2 an 1

eroberogeday lwm pln

- TMBr 124 -

KNitviTaCMuvijBiPBelak
!#> cUrRsayfaRtIekaN ABC CaRtIekaNEkgluHRtaEt
sin A sin B sin C
2 .
cos A cos B cos C
!$> cUrkMNt;RKb;KUncMnYnKt;viCman (a,b,c) ebIeKdwgfa
a 1 , b 1 CacMnYnbzm nig (a 1)(b 1) c 1 .
!%> eK[ n 4 CacMnYnKt;viCman nigyk a ,a , ....,a CacMnYnBitEdl
a a ... a 1 . cUrbgajfa
2

a1
a2
an
4
2

...

(a
a

...

a
a
)
1
1
n
n
a 22 1 a 23 1
a12 1 5

!^> eK[ a , b ,c CacMnYnBitviCmanmanplbUkesI # .


cUrRsayfa a b c ab bc ca .
!&> cUrRsaybBaak;fa

2|sin x cos x | 3 | sin x cos x | RKb; x (0, )


2
Edl n m CacMnYnKt;viCman .
!*> cUrkMNt;RKb;cMnYnKt;viCman a nig b edaydwgfa ab ba nig ba ba
CacMnYnKt;RBmKa .
!(> eK[ x , y , z CacMnYnBitviCmanEdl 1x y1 1z 1 . cUrRsayfa
x yz y zx z xy xyz x y z .
n

eroberogeday lwm pln

- TMBr 125 -

KNitviTaCMuvijBiPBelak
@0> eK[ a , b , c CabIcMnYnBitviCmanEdl abc 1 .
abc

a2 b 2 c2
3

cUrbgajfa
@!> eK[ a , b , c CabIcMnYnBitxusKa .
a2
b2
cUrRsayfa

2
2
(b c )

(c a )

c2
2
2
(a b )

@@> eK[ a , b , c CabIcMnYnBitminGviCmanEdl a2 b2 c2 a b c


cUrRsayfa a2b2 b2c2 c2a2 ab bc ca .
@#> eK[ a , b , c CabIcMnYnBitminGviCman . cUrRsayfa
(a 2 ab b 2 )(b 2 bc c2 )(c2 ca a 2 ) (ab bc ca)3

@$> eK[ a , b , c CabIcMnYnBitminGviCmanEdl abc 1 . cUrRsayfa


c a3 b 3 a b 3 c3 b c3 a3
3

.
2
2
2
2
2
2
2
a b
b c
c a
@%> eK[ a , b , c CabIcMnYnBitviCman . cUrRsaybBaak;vismPaB
a3
b3
c3

3
1
3
3
3
3
3
a (b c )
b (c a )
c (a b )

@^> cMeBaHRKb;cMnYnBitviCman a , b , c cUrRsayfa


a
b
c
3

ab
bc
ca
2

eroberogeday lwm pln

- TMBr 126 -

KNitviTaCMuvijBiPBelak
@&> cMeBaHRKb;cMnYnBitminGviCman x , y , z cUrRsayfa
2(1 x 2 )(1 y 2 )(1 z 2 ) (1 x )(1 y )(1 z )(1 xyz )

@*> cMeBaHRKb;cMnYnBitminGviCman a, b, c, d cUrRsayfa


4(1 a a 2 )(1 b b 2 )(1 c c 2 )(1 d d 2 ) (1 abcd )2

@(> ebIsmIkar x4 ax3 2x2 bx 1 0 manbsy:agticmYy


CacMnYnBitenaHbgajfa a4 b4 8 .
#0> cUrRsaybBaak;fa
a
b
c
9

(b c)2 (c a )2 (a b )2 4(a b c)

cMeBaHRKb; a , b , c CacMnYnBitviCman .
a3 b 3 c3 a2 b 2 c2
#!> cUrRsaybBaak;fa 2 2 2 b c a
b
c
a
cMeBaHRKb; a , b , c CacMnYnBitviCman .
#@> RKb;cMnYnBitviCman a , b , c cUrRsaybBaak;fa

bc ca ab
b
c
a

a
b
c
b c c a a b
##> eK[ a , b , c CabIcMnYnBitviCmanEdl abc 1 .

cUrRsayfa (a b)(b c)(c a) 4(a b c 1)


#$> cUrbgajfa (a2 2)(b2 2)(c2 2) 9(ab bc ca)
cMeBaHRKb;cMnYnBitviCman a , b , c .
eroberogeday lwm pln

- TMBr 127 -

KNitviTaCMuvijBiPBelak
#%> eKyk a , b , c CabIcMnYnBitviCman . cUrbgajfa
(b c a ) 2
(c a b )2
(a b c )2 3

2
2
2
2
2
2
5
(b c ) a
(c a ) b
(a b ) c

#^> eK[ x , y , z , t CabYncMnYnBitviCman . cUrRsayfa


2

x y z t



1 .
x y y z z t t x
#&> eK[ a , b , c nig x , y , z CacMnYnBit . cUrRsaybBaak;fa
4(a 2 x 2 )(b 2 y 2 )(c 2 z 2 ) 3(bcx cay abz )2

#*> eK[bIcMnYnBit a 1 , b 1 , c 1 . cUrRsaybBaak;fa


1 a2
1 b2
1 c2
2 .

2
2
2
1 b c 1 c a 1 a b
#(> eK[bIcMnYnBitviCman a , b , c Edl abc 1 . cUrRsaybBaak;fa
1
1
1
3

.
3
3
3
8
(1 a )

(1 b )

(1 c)

$0> eK[bIcMnYnBitviCman a , b , c Edl


a 2 b 2 c 2 (a b c )2 4 .
cUrRsaybBaak;fa ab 12 bc 12 ca 12 3
(a b )
(b c )
(c a )

$!> eK[bIcMnYnBitviCman a , b nig c . cUrRsayfa


(a5 a 2 3)(b 5 b 2 3)(c5 c 2 3) (a b c)3

eroberogeday lwm pln

- TMBr 128 -

KNitviTaCMuvijBiPBelak
$@> cUrRsayfacMeBaHRKb;cMnYnBitviCman a , b , c eKmanvismPaB
1
1
1
1

.
3
3
3
3
3
3
a b abc b c abc c a abc abc
mm 1 nn 1
$#> eKyk a m n Edl m nig n CacMnYnKt;viCman .
m n
cUrRsayfa am an mm nn .

$$> eK[ x ; y ; z CabIcMnYnBitviCmanEdl xyz 1 .


cUrRsaybBaak;fa
x9 y 9
x x y y
6

3 3

y9 z9
y y z z
6

3 3

z 9 x9
z z x x
6

3 3

$%> eK[ a , b , c CacMnYnviCmanEdl ab bc ca 3 .


1
cUrbgajfa 1 a 1(b c) 1 b 1(c a) 1 c 1(a b) abc
2

$^> eK[ a nig b CaBIrcMnYnBitminGviCmanEdl a

cUrRsayfa

b2 4

ab
2 1
ab2
x3 y 3 z 3
3
xyz | ( x y )( y z )( z x ) |
3
4
x;y;z0

$&> cUrRsayfa
cMeBaHRKb;
.
$*> eK[ a , b , c CaRbEvgRCugnRtIekaNmYy .
cUrRsayfa b aa c a bb c a bc c 3 .
eroberogeday lwm pln

- TMBr 129 -

KNitviTaCMuvijBiPBelak
$(> eK[ a , b , c CabIcMnYnBitEdlepgpat; a b c 3 .
cUrbgajfa | a | | b | | c | abc 4 .
%0> eK[ a , b , c CabIcMnYnBitviCmanEdl abc 1 .
cUrbgajfa 1 a b3 c ab bc6 ca .
%!> eK[ a 1 nig b 1 .
cUrbgajfa log a log b 2 log ( a b ) .
2
%@> eK[ x , y , z CabIcMnYnBitviCmanEdlepgpat; xyz 1 .
cUrbgajvismPaB
2

( x y 1) 2 ( y z 1) 2 ( z x 1) 2

xyz
z
x
y

%#>eK[ a , b , c CacMnYnBitviCman .
a 2b(b c) b 2c(c a) c 2a(a b )
cUrbgajfa a b b c c a 0 .
%$> eK[ a , b , c CabIcMnYnBitviCman . cUrbgajfa
a5 b 5 c5 (a b c)5 10
2

(
a

c
)
a 3 b 3 c 3 (a b c ) 3 9
%%> cMeBaHRKb;cMnYnBit x (0 , 4 )
cUrbgajfa (cos x)cos x (sin x)sin x .

eroberogeday lwm pln

- TMBr 130 -

KNitviTaCMuvijBiPBelak
%^> eK[ a , b , c , d CabIcMnYnBitEdlepgpat;vismPaB
(a 2 b 2 1)(c 2 d 2 1) (ac bd 1)2

cUrbgajfa a b 1 nig c d 1 .
%&> eK[ a , b , c CabIcMnYnBitviCmanEdl ab bc ca 1 .
cUrkMnt;tmGb,brmankenSam E a a b bb c c c a .
%*> eK[ x1 , x2 , ..., xn Edl n 2 CacMnYnBitviCmanEdlepgpat;
2

1
1
1
1

....

x1 1998 x 2 1998
xn 1998 1998

cUrbgajfa x1n.x2 1...xn 1998 .


%(> eK[ x , y , z CabIcMnYnBitviCmanEdl xyz x y z .
cUrRsayfa 1xzy 1yxz 1zyx 227
.
xyz
^0> eKyk a , b , c CacMnYnBitviCmanEdl ab bc ca abc
b 4 c4
c4 a4
a4 b4
cUrbgajfa ab(a3 b3 ) bc(b3 c3 ) ca(c3 a3 ) 1 .
^!> eKyk x , y , z CacMnYnBitviCmanEdl xyz 1
cUrbgajfa yz1 z zx1 x xy1 y 32 .
n

eroberogeday lwm pln

- TMBr 131 -

KNitviTaCMuvijBiPBelak
^@> cMnYnBit a, , b , c , x , y , z epgpat; a b c 0
nig x y z 0 . cUrbgajfa
a2x2
b2y 2
c2z 2
3

(by cz )(bz cy ) (cz ax )(cx az ) (ax by )(ay bx ) 4


3 3 xyz
x
y
z

1 x 1 y 1 z 1 3 xyz

^#> cUrbgajfa
cMeBaHRKb; 0 x , y , z 1 .
^$> cUrbgajfacMeBaHRKb; a , b , c CacMnYnBitviCmaneK)an
a
b
c

1 .

2
2
2
a 8bc

b 8ca

c 8ab

bgajfa 2 a 2 b 2 c 1
a 8bc
b 8ca
c 8ab
^%> cUrbgajfacMeBaHRKb; a , b , c CacMnYnBitviCmaneK)an
(a 1)(b 1)2
3 c a 1
3

2 2

(b 1)(c 1)2
3 a b 1
3

2 2

(c 1)(a 1)2

3 b c 1
3

^^> eK[ x , y , z CacMnYnBitviCmanEdl


cUrRsayfa
x 2 yz
2x ( y z )
2

y 2 zx
2y ( z x )

eroberogeday lwm pln

2 2

abc3

x y z 1

z 2 xy
2z ( x y )
2

- TMBr 132 -

KNitviTaCMuvijBiPBelak
^&> ebIcMnYnBitviCman a , b , c epgpat; a2 b2 c2 3
enaHcUrbgajfa
a2
b2
c2
(a b c ) 2

2
2
2
12
2bc
2ca
2ab

.
^*> eKmancMnYnBitviCman a , b , c epgpat; a b c 1 .
b ca c ab 3
cUrbgajfa aa bc
.

bc b ca c ab 2
^(> eK[ a , b , c Ca cMnYnBitviCman.
cUrbgajfa a 1 bc b 1 ca c 1 ab 12 ( ab1 bc1 ca1 ) .
&0> eK[ a , b , c , d CacMnYnBitviCman .
2

a4
b4
c4
d4
S

2
2
2
2
2
2
(a b)(a b ) (b c)(b c ) (c d)(c d ) (d a)(d 2 a 2 )

cUrbgajfa S a b 4 c d .
&!> eK[cMnYnBitviCman x1 , x2 , ...., xn ehIyeKtag
Sn x1 x 2 ... xn . cUrRsayfa
S2
Sn
(1 x1 )(1 x 2 )....(1 xn ) 1 S ...
2!
n!
&@> cMeBaHRKb;cMnYnKt;viCman n nig cMeBaHRKb;cMnYnBitviCman

epgpat; a1a 2a 3 ....an 1 .


n
ai
1 n 1
cUrbgajfa 4 2 a .

a1 , a 2 , ...., a n

i 1

eroberogeday lwm pln

ai 3

i 1 i

- TMBr 133 -

KNitviTaCMuvijBiPBelak
&#> eK[ a , b , c CacMnYnBitviCman edaydwgfa
1 1 1
a b c . cUrbgajfa
a b c
1
( 2a b c)

1
(a 2b c)

1
(a b 2c)

3
16

&$> smIkar x3 ax2 bx c 0 manbbICacMnYnBitviCman

mincaM)ac;xusKa .
cUrkMNt;tmGb,brmaEdlGacn 13a2abbc bc .
&%> cUrRsayfa
2
2 2
2

(
a
b
)(
a
ab
b
) 2 2 2 2 1
1
1
4
(
a
b
c
)(
)

2
3
ab bc ca
Cyc

cMeBaHRKb;cMnYnBitviCman a , b , c .
&^> ebI a , b , c CargVas;RCugnRtIekaNmYy ehIy r CakaMrgVg;
carwkkgnRtIekaNenaHcUrRsayfa a12 b12 c12 41r 2 .
&&> eK[ a , b , c CacMnYnBitviCmanehIyepgpat;lkxN
1 1 1
16(a b c) . cUrRsaybBaak;fa
a b c
1
(a b 2a 2c)3

eroberogeday lwm pln

8
1

(b c 2a 2b)3 (c a 2b 2c)3 9
1

- TMBr 134 -

KNitviTaCMuvijBiPBelak
&*> eK[ a , b , c CacMnYnBitviCmanEdl a b c 1 .

cUrbgajfa ab 2 bc 2 ca 2 83 .
1 a
1b
1 c
&(> eK[ a , b , c CabIcMnYnBitminGviCman nig x , y , z
CabIcMnYnBitviCmanedaydwgfa a b c x y z .
a3 b 3 c3
cUrRsaybBaak;fa 2 2 2 a b c .
x

*0> eK[ a , b , c 0 . cUrRsaybBaak;fa


a 3 4(b 3 c 3 ) b 3 4(c 3 a 3 ) c 3 4( a 3 b 3 ) 9

ca
2
bc
ab
*!> eKeGaybIcMnYnBitviCman a , b , c . cUrRsayfa
a3
b3
c3
3

(b c )3 (c a )3 (a b )3 8

*@> cMeBaHRKb;cMnYnBitviCman a , b , c eKkMNt;tag A a b3 c


3
.
1 1

a b c
3
A
cUrRsayfa G 14 43 . HA ?

G 3 abc

nig H 1

eroberogeday lwm pln

- TMBr 135 -

KNitviTaCMuvijBiPBelak
*#> eK[cMnYnBit x1 , x2 , y1 , y 2 , z1 , z 2 epgpat; x1 0 , x2 0
x1y1 z12 0 nig x 2y 2 z 2 2 0 .
cUrRsaybBaak;fa
8
( x1 x 2 )( y1 y 2 ) ( z1 z 2 )

1
x1y1 z1

1
x2y 2 z 2

(*)

*$> eK[RtIekaN ABC mYymanmMukgCamMuRsYc . cUrRsayfa


3 2
.
cos A cos B cos C
2
*%> eK[ x ; y ; z CacMnYnBitviCmanEdl xyz 1 . cUrRsayfa

1
1
1
1

( x 1)2 y 2 1 ( y 1)2 z 2 1 ( z 1)2 x 2 1 2

*^> eK[ a ; b ; c CacMnYnBitviCmanEdl abc 1 .


cUrRsayfa
.
*&> eK[ a1 , a2 , ..., an 0 nig x1 , x2 , ..., xn 0 .
cUrRsaytamkMeNInfa
a
a
a
(a a ... a )
.

...

x
x
x
x x .... x
a(b 2 b ) b(c 2 c ) c(a 2 a ) 0

**> eK[ a ; b ; c CabIcMnYnBitviCmanEdl abc 1 .


eroberogeday lwm pln

- TMBr 136 -

KNitviTaCMuvijBiPBelak
cUrbgajfa
1
1
1
3

a 3 (b c ) b 3 (c a ) c 3 (a b ) 2

*(> eK[ x ; y ; z 0 . cUrRsaybBaak;fa


x
y
z
1

x 2y 3z y 2z 3x z 2x 3y 2

(0> eK[sIVt a1 ; a2 ; ....; an epgpat;lkxN


a1 0 ; | a 2 || a1 1 |; ... nig | an || an 1 1 | .
cUrbgajfa a a n ... a 12 .
(!>eK[ a ; b ; c CabIcMnYnBitviCman .
cUrbgajfa
a
b
c
abc

) .
1 1 1 2 (1 3
b
c
a
abc

(@>eK[ a ; b ; c CaRbEvgRCugrbs;RtIekaNmYy .
cUrbgajfa

abc bca cab a b c

(#>eK[RtIekaN ABC mYymanmMukgCamMuRsYc . cUrbgajfa


sin 2 A sin 2 B sin 2 C 2 3 sin A sin B sin C .
($>eK[ A ; B ; C CamMukgrbs;RtIekaN ABC mYy .
eroberogeday lwm pln

- TMBr 137 -

KNitviTaCMuvijBiPBelak
cUrbgajfa cot A2 cot B2 cot C2 3 3
(%>eK[ A ; B ; C CamMuRsYckgrbs;RtIekaN ABC mYy .
cUrbgajfa (1 tan A)(1 tan B)(1 tan C) (1 3 ) .
(^>eK[ a ; b ; c CabIcMnYnBitviCman . cUrRsayfa
(a 2)(b 2)(c 2) 9(ab bc ca) .
(&>RbsinebI xyx (1 x)(1 y )(1 z ) Edl 0 x ; y ; z 1
cUrbgajfa x(1 z ) y(1 x) z(1 y ) 43 .
(*>eK[ a ; b ; c CaRbEvgRCugrbs;RtIekaNmYyEdlman
brimaRtesI 2 .
cUrRsayfa 32 a b c 2abc 2 .
((>eK[ x ; y ; z CabIcMnYnBitviCmanEdl x y z 1 .
cUrRsaybBaak;fa 1x 1 y1 1 1z 1 8 .

!00> eK[ a ; b ; c CabIcMnYnBitviCmanEdl a b c 1


1
1
2(a 3 b 3 c 3 )
1
bgajfa a2 b2 c2 3
abc
3

!0!> eK[ a , b , c CabIcMnYnBitviCman . cUrRsaybBaak;fa


eroberogeday lwm pln

- TMBr 138 -

KNitviTaCMuvijBiPBelak
a2
2a2 ab b2

b2
2b2 bc c2

c2
2c2 ca a2

abc
2

!0@> eK[bIcMnYnBitviCman a , b , c . cUrRsaybBaak;fa

29a 3 b 3 29b 3 c 3 29c3 a 3

4(a b c)

2
2
2
ab 6a
bc 6b
ca 6c
!0#> eK[ a , b , c CabIcMnYnBitviCman . cUrRsaybBaak;vismPaB
b3
c3
abc
a3

3
a 2 ab b 2 b 2 bc c 2 c 2 ca a 2
!0$> eK[bIcMnYnBitviCman a , b , c Edl a b c 1 .

cUrRsaybBaak;fa
1
1
1
1

ab 2c2 2c bc 2a2 2a ca 2b2 2b ab bc ca


!0%> eK[RtIekaN ABC mYymanRCug BC a , CA b , AB c

ehIymMukg A , B , C CamMuRsYcbmMuEkg .
tag S CapRkLan ABC
cUrRsaybBaak;fa 14 14 14 9 2 .
a

!0^> cUrbgajfa

16 S
1
1
1
4

( x y )2 ( y z )2 ( z x )2 xy yz zx

cMeBaHRKb;cMnYnBitminGviCmanxusKa x , y , z .

eroberogeday lwm pln

- TMBr 139 -

KNitviTaCMuvijBiPBelak
!0&> cUrbgajfa

1
1 1

a b

1 1
1
1

c d
ac bd
cMeBaHRKb;cMnYnBitviCman a , b , c , d .

!0*> eK[ a , b , c CacMnYnBitviCman . cUrRsaybBaak;fa


( 2a b c)2
( 2b c a )2
( 2c a b )2
2
2
8
2
2
2
2
2a (b c)
2b (c a )
2c (a b )

!0(> eK[ a , b , x , y , z CacMnYnBitviCman .


cUrbgajfa ay x bz az y bx ax z by a 3 b
!!0> eK[ n 2 CacMnYnKt; ehIy x1 , x2 , ...., xn CacMnYnBitviCman
edaydwgfa x x .... x 1 . cUrkMNt;tmtUcbMputn
2

x1
x2
xn

...
x 2 x 3 ... xn x 3 ... xn x1
x1 x 2 ... xn1

!!!> eK[ n CacMnYnKt;viCman . cUrkMNt;tmGb,brmanplbUk


2
3
n
xn
x2
x3

.....
.
S x1
n
2
3
(Poland 1995 )

!!@> cUrbgajfacMeBaHRKb;cMnYnBitviCman a , b , c , d eKman


a4b b 4c c4d d 4a abcd(a b c d )

eroberogeday lwm pln

- TMBr 140 -

KNitviTaCMuvijBiPBelak
!!#> eK[ a , b , c CacMnYnBitviCman . cUrbgajfa
2
3

2
3

2
3

2a
2b
2c



3
b c c a a b
(MOP 2002 )

!!$> cUrbgajfacMeBaHRKb;cMnYnBitviCman a , b , c eKman


a2
b2
c2
3

(a b )(a c) (b c)(b a ) (c a )(c b ) 4

!!%> cUrbgajfacMeBaHRKb;cMnYnBitviCman a , b , c eKman


a 2b b 2c c 2a

3


a 2c b 2a c 2b
!!^> eK[ a , b , c , x , y , z CacMnYnBitviCmanEdl x y z 1
3

cUrbgajfa
ax by cz 2 (ab bc ca)( xy yz zx ) a b c

!!&> eK[ a , b , c CacMnYnBitviCmanEdl abc 1 .


cUrbgajfa 5 ba bc ac (1 a)(1 b)(1 c)
!!*> eK[ a , b , c CacMnYnBitviCman . cUrbgajfa
a3
b3
c3
1

2 2

2
2
2
2
2
2
2
2
2
2
(2a b )(2a c ) (2b c )(2b a ) (2c a )(2c b ) a b c

!!(> eK[ a , b , c CacMnYnBitviCman . cUrbgajfa


(a5 a 2 3)(b 5 b 2 3)(c5 c 2 3) (a b c)3

eroberogeday lwm pln

- TMBr 141 -

KNitviTaCMuvijBiPBelak
a2 b 2 c2 a b c

b 2 c2 a2 b c a

!@0> RKb; a , b , c 0 cUrRsaybBaak;fa


!@!> cUrRsayfaRKb; a , b , c 0 eKman

b 2 c2 a 2 c2 a2 b 2 a 2 b 2 c2 3

a( b c )
b(c a )
c(a b )
2

!@@> eK[ a , b , c 0 Edl a b c 1 . cUrRsayfa


ab c bc a ca b 1 ab bc ca
!@#> eK[ a , b , c 0 Edl abc 8 . cUrRsayfa
a2
(a 3 1)(b 3 1)

b2
(b 3 1)(c3 1)

c2
(c3 1)(a 3 1)

4
3

!@$>cUrRsayfaRKb;cMnYnBit 0 x1 , x2 , ....., xn 12 eKmanvismPaB


n

n 1

1 1
i 1 x i
x1 x 2 ... xn

!@%> bgajfaRKb; a , b , c 0 eK)an

a2
b2
c2
1

( 2a b )( 2a c) ( 2b c)( 2b a ) ( 2c a )( 2c b ) 3

!@^> bgajfaRKb; a , b , c 0 eK)an


3

4a 2
4b 2
4c2

4a 4b 4b 4c 4c 4a
ab bc ca
3

eroberogeday lwm pln

- TMBr 142 -

KNitviTaCMuvijBiPBelak
!@&> eKman a , b , c , x , y , z IR Edlepgpat;smPaB
(a b c)( x y z ) 3

nig (a b c )(x y z ) 4
cUrbgajfa ax by cz 0 .
!@*> eK[ a , b , c 1 Edl a21 1 b21 1 c2 1 1 1 .
2

cUrRsayfa a 1 1 b 1 1 c 1 1 1 .
!@(> bgajfaRKb; a , b , c 0 eKman
a

a2 b2
b 2 c2
c2 a2
!#0> eK[ n 2 CacMnYnKt;viCman .

3 2
2

cMeBaHRKb;cMnYnBitviCman a1 , a2 ,..., an Edl a1 .a2 ....an 1


cUrRsaybBaak;fa
a1 1
a 1
a 1
2
... n
a1 a 2 ... an
2
2
2
!#!> eK[ a , b , c , x , y , z IR Edl xy yz zx 3 .
cUrbgajfa a(bycz ) b(czax) c(axby ) 3 .
!#@> eK[ a , b , c 0 Edl a b c 3 .
cUrbgajfa aba 1 bcb 1 cac 1 32 .
2

eroberogeday lwm pln

- TMBr 143 -

KNitviTaCMuvijBiPBelak
!##> eK[ a1 , a2 , ...., an 0 . cUrRsaybBaak;fa
a1 a1a2 3 a1a2a3 ... n a1a2...an n a1 a2 a1 a2 ... an
...
a1.
n
2
n
!#$> cUrRsayfaRKb; a , b , c 0 eKman
ab bc ca bc ca ab ca ab bc 3

2
2
2
2
2
2
2
b c
c a
a b
!#%> eK[ a , b , c CacMnYnBitviCman . cUrbgajfa
a 2 bc b 2 ca c 2 ab b c a

b 2 ca c 2 ab a 2 bc a b c
!#^> bgajfaRKb;cMnYnBitviCman x , y , z Edl x y z 1

eKman

x ( y z )2 y ( z x )2 z ( x y )2 3

!#&> eK[ a , b , c CacMnYnBitminGviCman nig KanBIrcMnYnNaesIsUn.


ca bc ca ab ca ab bc 3

cUrRsayfa abc2 bc
2
a2
a2 b2
b 2 c2
!#*> cMeBaHRKb;cMnYnBitviCman a nig b cUrRsayfa
a b
2ab
1 2
2 2
2
b a
a b
!#(> eK[ a , b , c CargVas;RCugnRtIekaNmYy .
cUrRsayfa 3a cb c 3b ac a 3c ba b 1

eroberogeday lwm pln

- TMBr 144 -

KNitviTaCMuvijBiPBelak
!$0> eK[ a , b , c CabIcMnYnBitviCman .
cUrRsaybBaak;fa

a 2c
b 2a 2 c 2b
b2 .
c .
a 2 b 2 c2
3b
3c
3a
!$!> cMeBaHRKb;cMnYnBitviCman x , y , z cUrRsayfa
27
( x 2 yz )2
xy( xy z 2 )( zx y 2 )
8
!$@> eK[ a , b , c CacMnYnBitviCmanEdl abc 1 .
a2 .

a 3 b 3 c3 3
a
b
c

cUrRsayfa

4
bc ca ab
!$#> eK[ a , b , c CacMnYnBitviCmanEdl a b c 2 .
cUrRsayfa b(aa b) c(bb c) a(ac c) 2

!$$> RKb;cMnYnBitviCman a , b , c cUrRsayfa


a 2 ac
b 2 ba
c 2 cb
abc

2b a c 2c a b 2a b c
2
!$%> eK[ a , b , c IR . cUrRsayfa
a 2 ab b 2 b 2 bc c 2 c 2 ca a 2 9

2
2
2
4
(a b )
(b c )
(c a )

eroberogeday lwm pln

- TMBr 145 -

KNitviTaCMuvijBiPBelak
!$^> eK[cMnYnBitviCman a , b , c Edl a b c 1 .
cUrRsayfa
a
b
c
1

4b 3bc 4c 4c 3ca 4a 4a 3ab 4b 3


!$&> RKb;cMnYnBitviCman a , b , c Edl a b c 3 .
cUrRsayfa b ac 1 c ba 1 a bc 1 3
!$*> RKb;cMnYnBitviCman a , b , c Edl a b c 3 .

cUrRsayfa
a
b
c
3

2b 3c 1 2c 3a 1 2a 3b 1 4
!$(> eK[ a , b , c 0 . cUrRsaybBaak;fa
a3 b3
b 3 c3
c3 a3
6(ab bc ca)

(a b c) (a b)(b c)(c a)
a2 b 2
b2 c2
c2 a2

!%0> ebI a , b , c CacMnYnBitviCmanenaHcUrRsayfa


a 2 (b c ) b 2 (c a ) c 2 (a b )
2
2
abc
2
2
2
2
b c
c a
a b
!%!> ebI a , b , c CacMnYnBitviCmanenaHcUrRsayfa
a b c 9(a 2 b 2 c 2 )

b c a
(a b c )2

eroberogeday lwm pln

- TMBr 146 -

KNitviTaCMuvijBiPBelak
!%@> ebI a , b , c CacMnYnBitviCmanEdl ab bc ca abc 4
cUrRsayfa
1
1
1
3

(a 1)2 (b c) (b 1)2 (c a ) (c 1)2 (a b ) 8

!%#> ebI a , b , c CacMnYnBitviCmanEdl abc 1 . cUrRsayfa


1
1
1
3

(a 1)2 (b c) (b 1)2 (c a ) (c 1)2 (a b ) 8

!%$> eKman a , b , c , d CacMnYnBitminGviCman . cUrRsayfa


(a b c d ) 4 a(c d ) b(d a ) c(a b ) d(b c)
!%%> ebI a , b , c CacMnYnBitviCmanEdl abc 1 enaHcUrRsayfa
3

3 2
3 2
a2
b
c
3

bc
ca
ab
2
!%^> eK[ a , b , c CacMnYnBitviCman . cUrRsayfa
3

a2
b2
c2
2
2
1
2
2
2
2
a ab b b bc c c ca a
!%&> eK[ a , b , c CacMnYnBitviCman . cUrRsayfa
a b
c 1 1 1 1 ab
bc
ca

( 2 2 2

)
2
2
2
bc ca ab a b c 2 b c c a a b
!%*> ebI a , b , c CacMnYnBitminGviCman enaHcUrRsayfa
2(a 2 1)(b 2 1)(c 2 1) (a 1)(b 1)(c 1)(abc 1)

eroberogeday lwm pln

- TMBr 147 -

KNitviTaCMuvijBiPBelak
!%(> eK[ x , y , z 0 nig n IN . cUrRsayfa
( xn 3 xn 3)( y n 3 y n 3)( z n 3 z n 3) ( x y z )3

!^0> ebI a , b , c 1 cUrRsayfa


1 a2
1 b2
1 c2

2
2
2
2
1 b c 1 c a 1 a b

!^!> ebI a , b , c IR bgajfa a b c 1


!^@> ebI x , y , z CabIcMnYnBitviCmanenaHcUrRsayfa
4

a5 b 5 c5 1

xy
yz
zx
x
y
z

xy x2 y2 yz y2 z2 zx z2 x2 2x z 2y x 2z y

!^#> eK[ a , b , c CabIcMnYnBitminGviCman . cUrbgajfa


(a2 ab b2 )(b2 bc c2 )(c2 ca a2 ) (ab bc ca)3

!^$> eK[ a , b , c 0 Edl a b c 1 .


bgajfa ab cb bc ac ca ba 2 .
!^%> eK[GnuKmn_ f (x) x x Edl 0 , 0
1 .
k> RKb; x [ 0 , 1 ] bgajfa f (x) 0 .
x> RKb; u , v 0 Edl u v bgajfa u .v u v .
2

eroberogeday lwm pln

- TMBr 148 -

KNitviTaCMuvijBiPBelak
!^^> eK[ a , b , c , x , x , x , ..., x CacMnYnBitviCmanEdl
a b c 1 nig x .x .x .x .x 1 . cUrbgajfa
1

.
!^&> rgVg;carwkkgnRtIekaN ABC b:HRCug BC , CA , AB erogKaRtg;
AB
BC
CA
3
A , B , C .Rsayfa

AB
BC
CA
(ax1 bx1 c)(ax 2 bx 2 c)...(ax5 bx 5 c) 1
2

!^*> eK[ 0 x 2 . cUrRsayfa

1
1

1
1
2

sin n x
cos n x

, n IN *

!^(> eK[ a , b , c CaRCugnRtIekaNmYy . cUrRsaybBaak;fa


k> 3 bc ca ab) (a b c)2 4(bc ca ab)
abc
abc
(p 2
x> a2 b2 c2 36
) Edl p
35
p
2
K> 32 b a c c b a a c b 3
X> (p a)(p b)(p c) abc
8
!&0> RtIekaN ABC manRCug a , b , c ehIytag r CakaMrgVg;carwkkg
nig p CaknHbrimaRtnRtIekaN .
cUrRsayfa 1 2 1 2 1 2 12 .
(p a )

eroberogeday lwm pln

(p b )

(p c )

- TMBr 149 -

KNitviTaCMuvijBiPBelak
!&!> cUrRsaybBaak;fa
k> abc a2 (p a) b2 (p b) c2 (p c) 32 abc
x> bc(b c) ca(c a) ab(a b) 48(p a)(p b)(p c)
K> a3 (p a) b3 (p b) c3 (p c) abcp
Edl a , b , c CaRCugnRtIekaNmYy nig p CaknHbrimaRt .
!&@> ebI a , b , c CaRCugnRtIekaNmYyenaHcUrRsayfa
a 2b(a b ) b 2c(b c) c 2a(c a ) 0

etIeBlNaeTIbeK)ansmPaB .
!&#> eKtag a , b , c CaRCugnRtIekaNmYy ehIy p a b2 c .
cUrbgajfa
k> 64p3 (p a)(p b)(p c) 27a2b2c2
2p
1
1
1
2 2 2
x> abc
a

c
K> p 1 a p 1 b p 1 c p9
X> 154 pb ac pc ab ap bc 92
g> p p a p b p c 3p

eroberogeday lwm pln

- TMBr 150 -

KNitviTaCMuvijBiPBelak
!&$> eK[RtIekaN ABC mYymanRCug a , b , c . tag R CakaMrgVg;carwkkg
nRtIekaNenH .
k> Rsayfa a2 b2 c2 8R 2 (1 cos A cos B cos C)
x> edayeRbIRTwsIbTLibniccUrTajfa cos A cos B cos C 18 .
K> ebI A , B , C CamMuRsYcenaH
!&%> kgRtIekaN ABC mYycUrRsayfa
k> a cos A b cos B c cos C 2RS
x> sin A sin B sin C sin 2A sin 2B sin 2C

2 sin A sin B sin C

3 3
2

!&^> eK[RtIekaN ABC mYymanRCug a , b , c .


k> Rsayfa sin A2 2 abc rYcTajfa sin A2 sin B2 sin C2 18 .
x> tag r nig R CakaMrgVg;carwkkg nig carwkeRkAnRtIekaN ABC .
bgajfa sin A2 sin B2 sin C2 4rR rYcTajfa R 2r .
!&&> eK[RtIekaN ABC mYy .
cUrbgajfa cos A 2(cos B cos C) 2 ?
etIeBlNaeTIbsmPaBenHkayCasmPaB ?
eroberogeday lwm pln

- TMBr 151 -

KNitviTaCMuvijBiPBelak
!&*> kgRKb;RtIekaN ABC cUrRsayfa
2
cos A (cos B cos C) 1
2
!&(> eK[ x , y , z CacMnYnBitEdl xyz 0 .

RKb;cMnYnBit 0 .

cUrRsayfa x cos A y cos B z cos C 12 ( xyz yzx zxy )


Edl A , B , C CamMukgnRtIekaNmYy .
!*0> kgRKb;RtIekaN ABC cUrRsaybBaak;fa
k> cos2(A B) cos2(B C) cos2(C A) 24cosA cosB cosC
x> tan A2 tan B2 tan C2 3
K> tan2 A2 tan2 B2 tan2 C2 1
!*!> kgRKb;RtIekaN ABC cUrRsayfa
k> tan A2 tan B2 5 tan B2 tan C2 5

C
A
tan tan 5 4 3
2
2

x> tan6 A2 tan6 B2 tan6 C2 19


K> cot A cot B cot C 3
!*@> kgRKb;RtIekaN ABC cUrRsayfa
k> cot A2 cot B2 cot C2 3 3
eroberogeday lwm pln

- TMBr 152 -

KNitviTaCMuvijBiPBelak
x> cot 2 A2 cot 2 B2 cot 2 C2 9
!*#> kgRKb;RtIekaN ABC cUrRsayfa
1 cos A cos B cos C
3
sin A sin B sin C

etIeK)ansmPaBenAeBlNa ?
!*$> eK[RtIekaN ABC mYymanRCug a , b , c .
RsayfaebI a b 2 c enaH A B 2 C .
!*%> eK[RtIekaN ABC mYymanRCug a , b , c .
tag R CakaMrgVg;carwkkg nig S CapRklarbs;RtIekaNenH .
cUrRsaybBaak;fa a sin A b sin B c sin C 2 R3S
!*^> RtIekaN ABC mYymanmMu A B C .
tag R CakaMrgVg;carwkkg ehIy r CacmayrvagpitrgVg;carwkkg nig pit
rgVg;carwkeRkAnRtIekaN . cUrRsaybBaak;fa
2R cos A R d 2R cos B R d 2R cos C
!*&> eK[RtIekaN ABC mYymanRCug a , b , c .

tag R CakaMrgVg;carwkkgehIy r CakaMrgVg;carwkeRkAnRtIekaN .


cUrRsaybBaak;fa 9r a sin A b sin B c sin C 92R .
eroberogeday lwm pln

- TMBr 153 -

KNitviTaCMuvijBiPBelak
!**> eK[RtIekaN ABC mYymanRCug a , b , c .
tag S CapRklanRtIekaNenH . cUrRsaybBaak;fa
k> a2 b2 c2 4 3 S
x> a4 b4 c4 16S2
K> b2c2 c2a2 a2b2 16S2
X> 4 3S a 9abc
bc
g> (abc)2 ( 4S3 )3
!*(> eK[RtIekaN ABC mYymanRCug a , b , c .
tag S CapRklanRtIekaNenH . cUrRsaybBaak;fa
27(b 2 c 2 a 2 )2 (c 2 a 2 b 2 )2 (a 2 b 2 c 2 )2 (4S )6

!(0> eK[RtIekaN ABC mYymanRCug a , b , c .


a2 b 2 c2
tag S CapRklanRtIekaNenH ehIyyk u
2
nig v bc ca ab .
2
(u v )( 3u 5 v )
2 (u v )
.
Rsayfa
S
12
12
!(!> tag S CapRkLa nig a , b , c CaRCugnRtIekaNmYy .

RKb;

3 3 a n b n cn
n 0 bgajfa S

4
3

eroberogeday lwm pln

.
- TMBr 154 -

KNitviTaCMuvijBiPBelak
!(@> eK[RtIekaN ABC mYymanRCug a , b , c . eKtag r nig R
erogKaCakaMrgVg;carwkkg nig kaMrgVg;carwkeRkAnRtIekaN .
yk O CapitrgVg;carwkeRkA I CapitrgVg;carwkkg nig H CaGrtUsg;
nRtIekaN ABC . Rsayfa OH 2 9R 2 (a2 b2 c2 )
nig I H 2 2r 2 4R 2 cos A cos B cos C .
!(#> eKtag r nig R erogKaCakaMrgVg;carwkkg nig kaMrgVg;carwkeRkAnRtIekaN
mYyEdlman p CaknHbrimaRt . cUrRsayfa
k> 9r(r 4R ) 3p2 (4R r )2
x> 6r(r 4R ) 2p2 R 2 2(r 2R )2
K> 2p2 (2R r ) R(4R r )2
X> r(16R 5r ) p2 4R 2 4Rr 3r 2
!($> RtIekaN ABC mYymanRCug a , b , c eKtag p CaknHbrimaRt r nig R
erogKaCakaMrgVg;carwkkg nig kaMrgVg;carwkeRkAnRtIekaN. cUrRsayfa
k> p2 27r 2
x> 2p2 27rR
K> 36r 2 a2 b2 c2 9R 2
a2 b 2 c2
X> 3(2rR r )
r 2 2R 2
4
g> 36r 2 bc ca ab 9R 2
2

eroberogeday lwm pln

- TMBr 155 -

KNitviTaCMuvijBiPBelak
c> 5rR r 2 bc ca4 ab (r R )2
q> a(p a) b(p b) c(p c) 9Rr
C> abc 8rR 2 (12 3 16)Rr 2
Q> R3 a1 b1 1c 2r3
j> a1 b1 1c 2(3R 3r )
1

1
1
1
1

R 2 bc ca ab 4r 2
z> 4r 2 a abc
bc
D> abc (R 3 )3

d>

!(%> eK[RtIekaNsmBa ABC mYymanRCg BC a , AC b , AB c


eKtag p CaknHbrimaRt r CakaMrgVg;cawikkg R CakaMrgVg;cawikeRkA nig S
CapRkLarbs;RtIekaN ehIy rA , rB , rC tagerogKaCakaMrgVg;
carwkeRkARtIekaNkgmMuerogKa A , B , C .
cUrRsaybBaak;fa
a2
b2
c2
k> r r r r r r 4
B C
C A
A B
x> p a 4R 3rA

eroberogeday lwm pln

- TMBr 156 -

KNitviTaCMuvijBiPBelak
a 2 (b c)2 4R rA
3

2S
pa

K>
X> p2 rA 2 rB2 rC2
g> raA rbB rcC 3 8 3

rBrC rCrA rArB 3


c> bc ca ab 4
q> 9r rA rB rC 92 R
C> rA 2 rB2 rC2 274 R 2
Q> r 2 rA 2 rB2 rC2 7R 2

j> S 3 (R r )2
!(^> eK[RtIekaNsmBa ABC mYymanRCg BC a , AC b , AB c
eKtag p CaknHbrimaRt r CakaMrgVg;cawikkg R CakaMrgVg;cawikeRkA nig S
CapRkLarbs;RtIekaN ehIy ma , mb , mc tagerogKaCaemdan
nig ha , hb , hc Cakm<s;KUsecjBIkMBUlerogKa A , B , C .
cUrRsaybBaak;fa
k> 9r ma mb mc 92 R
x> 9r ha hb hc 92R
eroberogeday lwm pln

- TMBr 157 -

KNitviTaCMuvijBiPBelak
3
K> S 27 (ma mb mc )2
X> ma2 mb 2 mc2 3 3S

g> ma mb mc 4R r
!(&> eK[RtIekaNsmBa ABC mYymanRCg BC a , AC b , AB c
tag A , B , C erogKaCaknHbnat;BuHkgnmMu A , B , C nig S Ca
pRkLanRtIekaN . cUrRsayfa
k> A 2 B 2 C2 3 3S
x> 16( A4 B4 C4 ) 9(a4 b4 c4 )
K> A . B . C p.S Edl p a b2 c .
!(*> eKyk P CacMnucenAkgRtIekaN ABC ehIy D , E , F CaRbsBVrvag
AP , BP , CP CamYyRCug BC , CA , AB erogKa .
BE CF 9

.
cUrRsayfa AD
AP BP CP 2
!((> rgVg;carwkkgnRtIekaN ABC b:HnwgRCug BC , CA , AB erogKaRtg;
cMNuc D , E , F .
2

CA AB
Rsayfa BC


12
EF FD DE

eroberogeday lwm pln

- TMBr 158 -

KNitviTaCMuvijBiPBelak
@00> eK[RtIekaN ABC mYymankm<s; AD , BE , CF .
2

EF FD DE
3
Rsayfa BC



4
CA AB

@0!> eKmanRtIekaN ABC mYymanRCug a , b , c nigpRkla S

tag T CapRkLanRtIekaNsmgScarwkkgRtIekaN ABC xagelI .


2 3S 2

cUrRsayfa T 2 2 2
.
a b c 4 3S
@0@> eKmanRtIekaNBIr ABC nig XYZ carwkkgrgVg;EtmYy .
eKdwgfa AX // BC , BY // CA , CZ // AB .
tag a , b , c CaRCugn ABC nig x, y, z CaRCugn XYZ .
Rsayfa xa by cz 1 nig ax by zc 3 .
@0#> eK[RtIekaN ABC mYyEkgRtg; A nigmankm<s; AH h .
tag r CakaMrgVg;carwkkgnRtIekaN .
cUrRsayfa 2 1 hr 12 ?
@0$> eK[RtIekaN ABC mYyEkgRtg; A .
eKtag r CakaMrgVg;carwkkgnRtIekaN . cUrRsaybBaak;vismPaB
2
BC
r
AB.AC
.
etI
e
BlNaeTI
b
eK)ansmPaB
?
2
2
eroberogeday lwm pln

- TMBr 159 -

KNitviTaCMuvijBiPBelak
@0%> eKtag P CacMnucmYyenAkgRtIekaN ABC ehIyyk D , E , F CacMnuc
RbsBVrvagbnat; AP , BP , CP CamYyRCug BC, CA, AB erogKa .
BP CP
AP BP CP
cUrRsayfa AP

6 nig
.
.
8
PD PE PF
PD PE PF
@0^> eKtag A1 , B1 , C1 CabIcMnucsitenAelIRCug BC , CA , AB n ABC
1 BB1 CC1 3 .
cUrbgajfa 12 AA
AB BC CA
2
@0&> kgRtIekaN ABC mYycUrRsayfa
r
cos 2 A cos 2 B cos 2 B cos 2 C cos 2 C cos 2 A
1

R
cos A cos B
cos B cos C
cos C cos A

@0*> eKmanRtIekaN ABC mYyEkgRtg; A . tag M , N , P CacMnucb:H


rvagrgVg;carwkkgCamYyRCug BC, CA, AB . cUrRsayfa
4 3
MN MP
BC .
9
@0(> RtIekaN ABC mYycarwkkgrgVg;pit O kaM R .
tag R1 , R 2 , R 3 CakaMrgVg;carwkeRkAnRtIekaN OAB, OBC, OCA
erogKa . Rsayfa R1 R1 R1 3 R2 3 ?
1

@!0> ebI ABCD CaRbelLRkamenaHbgajfa | AB2 BC2 | AC.BD

eroberogeday lwm pln

- TMBr 160 -

KNitviTaCMuvijBiPBelak
@!!> ebI a , b , c CaRbEvgRCugnRtIekaNmYy ehIy ma , mb , mc Caemdan
nig R CakaMrgVg;carwkkgenaHbgajfa
k>

a2 b 2 b 2 c2 c2 a2

12R
mc
ma
mb

x> ma (bc a2 ) mb (ca b2 ) mc (ab c2 ) 0


@!@> (APMO, 1996) tag a, b, c CaRbEvgRCugnRtIekaNmYy . cUrbgajfa
abc bca cab a b c
@!#> (IMO, 1964) tag a, b, c CaRbEvgRCugnRtIekaNmYy . cUrbgajfa
a 2 (b c a ) b 2 (c a b ) c 2 (a b c) 3abc

@!$> (IMO, 1983) tag a, b, c CaRbEvgRCugnRtIekaNmYy . cUrbgajfa


a 2b(a b ) b 2c(b c) c 2a(c a ) 0 .
@!%> eK[RtIekaN ABC mYyEkgRtg; A . cMeBaHRKb;cMnYnKt; n 2
cUrRsayfa ABn ACn BCn .
@!^> tag ha , hb , hc CaRbEvgkm<s;nRtIekaN ABC mYyEdlcarwkeRkA
rgVg;pit I nigkaM r . cUrRsayfa
k> 1r h1 h1 h1
a

x> ha hb hc 9r

eroberogeday lwm pln

- TMBr 161 -

KNitviTaCMuvijBiPBelak
@!&> (Korea, 1995)eK[RtIekaN ABC mYyehIytag L , M , N
CacMnucsitelIRCug BC , CA , AB erogKa . tag P , Q nig R
CacMNucRbsBVrvagbnat; AL , BM nig CN CamYyrgVg;carwkeRkAn
AL BM CN

9 .
ABC erogKa . cUrRsayfa
LP MQ NR
@!*> (Shortlist IMO, 1997)RbEvgRCugnqekaN ABCDEF epgpat;
AB BC , CD DE nig EF FA .
DE FA 3
cUrRsayfa BC

.
BE DA FC 2
@!(> eKyk AD , BE , CF Cakm<s;nRtIekaN ABC ehIy PQ , PR, PS
CacmayBIcMnuc P eTARCug BC, CA, AB erogKa .
AD BE CF
cUrbgajfa PQ

9 .
PR PS
@@0> sntfargVg;carwkkgnRtIekaN ABC b:HeTAnwgRCug BC, CA, AD
p2
erogKaRtg; D, E, F . bgajfa EF FD DE 3
Edl p CaknHbrimaRtn ABC .
2

@@!> RtIekaN ABC mYymanRCug a , b , c nigtag S CapRklan ABC


cUrRsaybBaak;fa
k> a2 b2 c2 (a b)2 (b c)2 (c a)2 4 3S
x> ab bc ca 4 3S
eroberogeday lwm pln

- TMBr 162 -

KNitviTaCMuvijBiPBelak
a b c)abc
K> 3(ab
4 3S
bc ca
@@@> tag a , b , c CaRCug nig R CakaMrgVg;carwkeRkAnRtIekaNmYy .
a2
b2
c2
cUrRsayfa b c a c a b a b c 3 3R .
@@#> ctuekaNe)a:g ABCD mYymanRCug a , b , c , d . tag S CapRkLa

rbs;ctuekaNenH . cUrRsayfa
k> S ab 2 cd

ac bd
2
K> S a 2 c b 2 d

@@$> ebI P CacMnucenAkgRtIekaN ABC

x>

Edl PA , m PB, n PC

enaHcUrbgajfa (m mn n)( m n) a2 b2m c2n


Edl a , b , c CaRCugn ABC .
@@%> eKtag O CapitrgVg;carwkkg nig G CaTIRbCMuTmn;n ABC mYy .
eKtag r nig R erogKaCakaMrgVg;carwkkg nig carwkeRkAnRtIekaN .
(Balkan, 1996)
cUrRsayfa OG R(R 2r ) .

eroberogeday lwm pln

- TMBr 163 -

KNitviTaCMuvijBiPBelak
@@^> (Taiwan, 1997) eK[RtIekaN ABC mYycarwkkgrgVg;pit O nigkaM R
RsayfaebI AO kat;rgVg;carwkeRkAn OBC Rtg; D ehIy BO
kat;rgVg;carwkeRkAn OCA Rtg; E ehIy CO kat;rgVg;carwkeRkAn
OAB Rtg; F enaH OD.OE.PF 8R 3 .
@@&> (APMO, 1997)eK[ ABC CaRtIekaNmYy . bnat;BuHkgnmMu A
CYbGgt; BC Rtg; X nigCYbrgVg;carwkeRkAn ABC Rtg; Y .
AX
tag La AY
.eKkMNt; Lb nig Lc tamrebobdUcKaEdr .
cUrRsayfa

La

Lb

Lc

?
sin A sin B sin C
@@*> (Balkan, 1999)eK[ ABC CaRtIekaNmYymanmMukgCamMuRsYc ehIytag
L , M , N CaeCIgncMeNalEkgBITIRbCMuTmn; G nRtIekaN ABC
eTAelIRCug BC, CA, AB erogKa .
cUrbgajfa 274 SSLMN 14 ?
2

ABC

@@(> tag D nig E CacMnucenAelIRCug AB nig CA nRtIekaN ABC


edaydwgfa DE Rsbnwg BC ehIy DE b:HnwgrgVg;carwkkgn
AB BC CA
.
ABC .cUrbgajfa DE
8
(Italy, 1999)

eroberogeday lwm pln

- TMBr 164 -

KNitviTaCMuvijBiPBelak
@#0> (Mediterranean, 2000) tag P, Q, R, S CacMnuckNal erogKa
BC, CD, DA, AB nctuekaNe)a:g ABCD mYy .
cUrRsayfa
4( AP2 BQ2 CR2 DS2 ) 5( AB2 BC2 CD2 DA2 )

@#!> eKtag R nig r CakaMrgVg;carwkeRkA nig kaMrgVg;carwkkgn ABC .


sntfa A CamMuFMCageKnRtIekaN ABC .
yk M CacMNuckNaln BC ehIy X CaRbsBVrvagbnat;b:H
eTAnwgrgVg;carwkeRkARtIekaN ABC Rtg; B nig C .
cUrRsayfa Rr AM
(Korea, 2004)
AX
@#^> eK[RtIekaN ABC mYyehIyyk O CacMNucmYyenAkgRtIekaNenH .
bnat; OA, OB, OC CYbRCugnRtIekaNRtg; A1 , B1 , C1 erogKa .
tag R1 , R 2 , R 3 CakaMrgVg;carwkeRkAnRtIekaN OBC, OCA, OAB
erogKaehIy R CakaMnrgVg;carwkeRkAn ABC .
1 .R OB1 .R OC1 .R R
cUrRsayfa OA
1
2
3
AA
BB
CC
1

#@&>mMukgnRtIekaN ABC mYyCamMuRsYc ehIy ACB 2 ABC .


D CacMnucmYysitelIRCug BC ehIyepgpat; 2BAD ABC
1
1
1

cUrRsayfa BD
?
AB AC
( Mathematical Olympiad in Poland 1999 )

eroberogeday lwm pln

- TMBr 165 -

KNitviTaCMuvijBiPBelak
@#*> cMNuc D nig E sitenAelIRCug BC nig AC nRtIekaN ABC erogKa
bnat; AD nig BC kat;KaRtg; P .
K nig L CacMNucenAelIRCug BC nig AC erogKaEdl CLPK
BD
CaRbelLRkam . cUrRsayfa AE

EL DK
(Mathematical Olympiad in Poland 2000 )

@#(> bIcMNucxusKa A , B , C sitenAelIrgVg; (O) .


bnat;b:HrgVg; (O) Rtg; A nig B kat;KaRtg; P .
bnat;b:HrgVg; (O) Rtg; C kat;bnat; AB Rtg; Q .
cUrRsaybBaak;fa PQ 2 PB 2 QC2 ?
Mathematical Olympiad in Poland 2002 )
@$0>bnat;b:HeTAnwgrgVg;carwkkgnRtIekaNsmgS ABC kat;RCug AB
nig AC Rtg; D nig E .
AE
cUrRsaybBaak;fa AD

1 ?
DB EC
( Mathematical Olympiad in Poland 1995 )

@$!> bnat;BuHkgnmMu A , B , C nRtIekaN ABC mYykat;RCugQmRtg;


D, E, F erogKa ehIykat;rgVg;carwkeRkAnRtIekaN ABC Rtg;
AD BE CF
K , L, M erogKa . bgajfa

9 ?
DK EL FM
( Mathematical Olympiad in Poland 1996 )

eroberogeday lwm pln

- TMBr 166 -

KNitviTaCMuvijBiPBelak
@$@> kgRtIekaN ABC mYyman AB AC . D CacMNuckNaln BC
ehIy E sitenAelI AC . P nig Q CaeCIgncMeNalEkgBIcMNuc
B nig E erogKaeTelIbnat; AD .
bgajfa BE AE AC luHRtaEt AD PQ ?
(Mathematical Olympiad in Poland 1997 )

@$#>eKtag O CapitrgVg;carwkeRkA nig H CaGrtUsg;nRtIekaN ABC mYy


EdlmanmMukgCamMuRsYc .
cUrRsayfapRkLamYynRtIekaN AOH, BOH, COH esInwgplbUk
npRkLaBIrepSgeTot ?
(AsianPacific Mathematical Olympiad 2004 )

@$$> kgRtIekaN ABC cMNuc M nig N sitelIRCug AB nig AC erogKa


edaydwgfa MB BC CN .
tag r nig R erogKaCakaMrgVg;carwkkgnig carwkeRkAnRtIekaN .
cUrsresrkenSampleFob MN
CaGnuKmn_n R nig r .
BC
(AsianPacific Mathematical Olympiad 2005)

eroberogeday lwm pln

- TMBr 167 -

KNitviTaCMuvijBiPBelak
@$%> eK[ ABC CaRtIekaNmYymanmMukgCamMuRsYc ehIy BAC 60o
nig AB AC . yk I CapitrgVg;carwkkg nig H CaGrtUsg;
n ABC . cUrRsayfa 2AHI 3ABC ?
(AsianPacific Mathematical Olympiad 2007)

@$^> eKtag G CaTIRbCMuTmn;nRtIekaN ABC ehIy M CacMNuckNaln


RCug BC . bnat;KUsecjBI G Rsbnwg BC kat; AB Rtg; X ehIy
Ktat; AC Rtg; Y . sntfa XC nig GB kat;KaRtg; Q ehIy YB
nig GC kat;KaRtg; P .
cUrRsayfaRtIekaN MPQ dUcKanwgRtIekaN ABC ?
(AsianPacific Mathematical Olympiad 1991 )

@$&> elIRCug AB nig BC nkaer ABCD eKedAcMNuc E nig F erogKa


Edl BE BF . tag BN Cakm<s;nRtIekaN BCE .
cUrRsayfa DNF 90o ?
(AustrianPolish Mathematical Competition 1979)

@$*> rgVg;carwkkgnRtIekaN ABC b:HRCug BC nig AC Rtg; D nig E


erogKa . bgajfaebI AD BE enaH ABC CaRtIekaNsm)at ?
(Austrian Mathematical Olympiad 2006)

eroberogeday lwm pln

- TMBr 168 -

KNitviTaCMuvijBiPBelak
@$(> kgRtIekaN ABC mYyeKyk E CacMNuckNalnRCug AC nig F
CacMNuckNalnRCug BC . tag G CacMeNalEkgBI C eTA AB .
bgajfa EFG CaRtIekaNsm)atluHRtaEt ABC sm)at .
(Austrian Mathematical Olympiad 2008)

@%0> eKyk D, E nig F erogKaCacMNuckNalnRCug BC, CA nig AB


nRtIekaN ABC . yk Ha , Hb , Hc CaeCIgncMeNalEkgBI
A, B, C eTARCugQmerogKa .yk P, Q, R CacMNuckNaln
Hb H c H c H a nig Ha H b erogKa .
bgajfa PD, QE nig RF RbsBVKaRtg;cMnucmYy ?
(Austrian Mathematical Olympiad 2009)

@%!>kgctuekaNe)a:g ABCD mYyeKyk E CacMnucRbsBVrvagGgt;RTUg


ehIy S1 , S2 nig S CapRkLan ABE , CDE nig ABCDerogKa
cUrRsayfa S1 S2 S ?
(Austrian Mathematical Olympiad 1990)

@%@> bBaekaNe)a:g ABCDE mYycarwkkgrgVg; . cmayBI A eTAbnat;


BC, CD, DE esIerogKa a, b, c .
cUrKNnacmayBIkMBUl A eTAbnat; (BE) .
(Austrian Mathematical Olympiad 1990)

eroberogeday lwm pln

- TMBr 169 -

KNitviTaCMuvijBiPBelak
@%#> eK[RtIekaNsm)at ABC mYyEdl A 90o .
eKyk M CacMNuckNaln AB . bnat;mYyKUsecjBI A ehIyEkg
nwg CM kat;RCug BC Rtg; P . bgajfa AMC BMP ?
(Baltic Way 2000)

@%$>eK[RtIekaN ABC mYymanmMu A 120o . cMNuc K nig L


sitenAelIRCug AB nig AC erogKa .
eKsg;RtIekaNsmgS BKP nig CLQ enAeRkARCugnRtIekaN ABC
cUrRsayfa PQ 23 ( AB AC) ?
(Baltic Way 2000)

@%%> kgRtIekaN ABC mYyeK[ 2AB AC BC . cUrRsayfapitrgVg;


carwkkg ABC pitrgVg;carwkeRkA ABC ehIycMNuckNaln
AC nig BC sitenAelIrgVg;EtmYy .
(Problems for the Team Competition Baltic Way 1999)

@%^>eK[ ABC nig DAC CaRtIekaNsm)atBIrEdlmanmuMkMBUl


BAC 20o nig ADC 100o .
cUrRsayfa AB BC CD ?
(Flanders Mathematical Olympiad 1996)

eroberogeday lwm pln

- TMBr 170 -

KNitviTaCMuvijBiPBelak
@%&>eKtag R CakaMrgVg;carwkeRkA nig r CakaMrgVg;carwkkgn ABC mYy.
S CacMnucmYyenAkg ABC ehIybnat; AS, BS, CS kat;RCugQm
Rtg; X, Y, Z erogKa .
CY.AY AZ.BZ R r
cUrRsayfa BX.CX

2
2
2
r
AX

BY

CZ

( Bosnia and Hercegovina Mathematical Olympiad 2000 )


o

@%*> eK[RtIekaNEkgmYy ABC Edl ACB 90 ehIyyk


CH Edl H AB CakMBs;cMeBaHRCug AB ehIytag P nig Q
CacMNucb:HrvagrgVg;carwkkgnRtIekaN ABC CamYyRCug AC nig BC
erogKa . ebI AQ HP cUrkMNt;pleFob AH
?
BH

(Bulgarian Mathematical Olympiad 2006)

@%(> eK[ctuekaNcarwkkgrgVg; ABCD mYy. bnat; AD nig BC CYbKa


Rtg;cMNuc E ehIyGgt;RTUg AC nig BD CYbKaRtg; F .
ebI M nig N CacMNuckNaln AB nig CD enaHcUrbgajfa
MN 1 AB CD

.
EF 2 CD AB
(Bulgarian Mathematical Olympiad 1997)

eroberogeday lwm pln

- TMBr 171 -

KNitviTaCMuvijBiPBelak
@^0> bBaekaNe)a:g ABCDE mYycarwkkgrgVg;kaM R . eKsMKal; rXYZ Ca
rgVas;kaMnrVg;carwkkgn XYZ . cUrRsaybBaak;fa
k> cos CAB cos ABC cos BCA 1 rABC
R
x> ebI rABC rAED nig rABD rAEC enaH ABC AED
(Bulgarian Mathematical Olympiad 1998)

@^!> eK[bnat; () kat;RCug AC nig BC nRtIekaN ABC erogKaRtg;


cMNuc E nig F .
cUrRsayfabnat; () kat;tampitrgVg;carwkkgnRtIekaN ABC
AE
BF
luHRtaEt BC. CE
AC.
AB .
CF
(Bulgarian Mathematical Olympiad 1973)

@^@> eKtag M CaTIRbCMuTmn;nRtIekaN ABC mYy .


cUrRsayfavismPaB sin CAM sin CBM

2
3

(Bulgarian Mathematical Olympiad 1997)

@^#> ebI a , b , c CaRCug nig R CakaMrgVg;carwkeRkAnRtIekaN ABC


enaHbgajfa R

a2 b2
2 2a 2b c
2

etIsmPaBmanenAeBlNa?
eroberogeday lwm pln

.
( Baltic Way 1998 )

- TMBr 172 -

KNitviTaCMuvijBiPBelak

ksaeyag
!>

101 problems in Algebra


( Titu Andreescu & Zuming Feng)

@>

Selected Problems of the Vietnamese Mathematical


Olympiad

(19622009)

( Le Hai Chau & Le Hai Khoi )

#>
$>

TOPICS IN INEQUALITIES

( Hojoo Lee )

Mathematical Olympiad in China


(Xiong Bin & Lee Peng Yee )

%>

Lecture Notes onMathematical Olympiad Courses


( Xu Jiagu )

^>

INFINITY

&>

(Hojoo Lee, Tom Lovering, and Cosmin Pohoat )


Secrets in Inequalities
(PHAM KIM HUNG)

eroberogeday lwm pln

- TMBr 173 -

You might also like

pFad - Phonifier reborn

Pfad - The Proxy pFad of © 2024 Garber Painting. All rights reserved.

Note: This service is not intended for secure transactions such as banking, social media, email, or purchasing. Use at your own risk. We assume no liability whatsoever for broken pages.


Alternative Proxies:

Alternative Proxy

pFad Proxy

pFad v3 Proxy

pFad v4 Proxy